You are on page 1of 147

Exam APM: Old SOA Exams related to QFIC syllabus (Fall 2014)

Also:

Reading relevance to QFIC Exam (Fall 2014)

List of relevant questions


6. (8 points) The implied volatility of options on the dividend-paying stock of
Company ZYX increased during the past few months. The following table
presents the current implied volatility for different strike prices of put options on
the Company stock.

Strike Price Implied Volatility


90 0.185
95 0.195
100 0.200
105 0.200
110 0.190
115 0.180

You are given the following information about a put option on Company ZYX
stock:

• Stock price 100


• Term 3 months
• Risk free rate 3% (continuously compounded)
• Quarterly dividend 0.40 (next ex-dividend date is in 1 month)

(a) Calculate the price of the at-the-money European put option according to
the Black-Scholes-Merton model.

(b) Explain the concept of volatility smile.

(c) Interpret the observed volatility curve.

(d) Criticize the Black-Scholes-Merton model assumptions.

(e) Recommend one alternative to the Black-Scholes-Merton pricing model


for options on this stock that can reflect a more realistic behavior of stock
prices.

EXAM APMV: Spring 2007 7 GO ON TO NEXT PAGE


Advanced Portfolio Management
Morning Session
10. (6 points) You are given the following statistics on two assets that could be used
to back a liability:

Return Return
Year Asset 1 Asset 2 Liability
(Asset 1) (Asset 2)
2001 100.0 100.0 90
2002 110.0 108.0 95 10.0% 8.0%
2003 121.0 113.4 100 10.0% 5.0%
2004 123.4 116.8 105 2.0% 3.0%
2005 125.9 122.6 110 2.0% 5.0%
2006 132.2 128.7 115 5.0% 5.0%
Mean 5.74% 5.18%
Standard
4.02% 1.79%
deviation

Assume that the risk-free rate is 4%.

(a) Calculate the Sharpe Ratio for these two assets.

(b) Describe the shortcomings of the Sharpe ratio as a measure for investment
decisions in an asset-liability framework.

(c) Calculate the Risk Adjusted Change in Surplus (RACS) for these two
assets.

(d) Explain how the RACS may be more effective in an asset-liability


framework.

EXAM APMV: Spring 2007 11 GO ON TO NEXT PAGE


Advanced Portfolio Management
Morning Session
23. (4 points) Assume f and g are two securities which depend on the same source of
uncertainty. In addition, assume that the securities provide no income during the
time period under consideration. The expected growth rate and volatility for f and
g are (m f , σ f ) and (m g , σ g ) respectively.

(a) Define the following concepts:

(i) martingale
(ii) numeraire
(iii) equivalent martingale measure
(iv) forward risk neutral

(b) Describe the process h = f/g using g as the numeraire in the forward risk
neutral world with respect to g.

**END OF EXAMINATION**
AFTERNOON SESSION

EXAM APMV: Spring 2007 12 STOP


Advanced Portfolio Management
Afternoon Session
6.
Learning Objectives:
1a) Demonstrate mastery of option pricing techniques

1c) Identify limitations behind various option pricing techniques.

1d) Describe how option pricing models can be modified or alternative techniques
that can be used to deal with option pricing techniques limitations

This is a synthesis question asking candidates to demonstrate mastery of the Black-


Scholes-Merton model, understanding of its limitations (especially as exemplified by
volatility smiles) and recommend an alternative pricing model for options. The best
sources for answering this question could be found in chapters 13, 16 and 24 of the Hull
“Options, Futures, and Other Derivatives” (6th Edition) textbook. Credit was also given
for answers based on other readings from the syllabus resources if relevant. Any
alternative model recommendation was acceptable as long as the candidate provided
appropriate background information and adequate justification.

Solution:
(a)
Option price P = Ke-rTN(-d2) - S0N(-d1)
Where S0 = current stock price adjusted for dividend

S0 = 100 – 0.40*e(-0.03/12) = 100 – 0.40*0.9975 = 99.6

N() is the cumulative prob. distribution function for standardized normal


distribution

K = strike price = 100 (at-the-money)


r = risk free rate = 3%
T = term = 0.25
d1 = (ln(S0/K) + (r+σ2/2)T)/σ√T
d2 = d1 - σ√T
σ = volatility = 0.2

So, d1 = (ln(99.6/100) + (.03+.04/2)*.25)/(.2*.5) = (-0.004 + 0.125)/0.1 = 0.0849


and d2 = 0.0849 - .2*.5 = -.015
N(-d1) = N(-0.0849) = 0.4662
N(-d2) = N(0.015) = 0.506
P = 100*e(-0.03*0.25)*0.506 – 99.6*0.4662
P = 3.788

(b)
• Volatility smile is a plot of implied volatility as function of strike price.
• Occurs because stock price changes are not really lognormally distributed.
6.
• Calculate from actual option prices in the market using the Black-Scholes
formula.
• The tails of the implied distribution are usually heavier than of the lognormal.
• So implied volatility is typically progressively higher as option moves into the
money or out of the money creating a “smile”.
• For equities, traders typically assume heavier left tail and a less heavy right
tail.
• So for equities the smile is usually downward sloping.
• Shape of smile depends on time to maturity.
• Volatility surfaces combine volatility smiles and volatility term structure.

(c)
• The observed example displays volatility frown rather than volatility smile.
• Volatility is lower for lower and higher strike prices.
• Volatility implied from option with strike 100 will overprice option with
strike 90 or 110.
• This is the situation when stock price distribution is bimodal.
• Volatility increase together with volatility frown may suggest that a jump in
stock price is anticipated but a direction is unpredictable.

(d)
Black-Scholes-Merton model makes the following more or less unrealistic assumptions:
1. The stock price distribution is lognormal with parameters μ and σ constant
2. Short selling with full use of proceeds is permitted
3. No transaction costs or taxes
4. Securities are perfectly divisible
5. No dividends during life of derivative
6. No riskless arbitrage
7. Continuous trading
8. Risk-free rate r is constant and same for all maturities

• Assumption regarding μ and r can be relaxed.


• For example, they can be known functions of t.
• Interest rates can even be stochastic.
• Traders overcome weakness of the model by using volatility surfaces to
account for changing σ.
• Easy adjustment to account for dividends.
• Short selling may sometimes be limited.
• Transaction costs and taxes may somewhat distort pricing.
• The fact that securities are not perfectly divisible may only be relevant for
small amounts.
• Trading is not always continuous – problems if stock prices jump or crash.
6.
(e)
Could use Implied Volatility Function (IVF) model.
Assumes stock price process dS = (r(t)-q(t))Sdt + σ(S,t)Sdz
Provides exact fit to the observed prices.
IVF model could fit to observed volatility.

Alternative model recommendations were also acceptable:

Binomial model might be used if large jump is expected.


Binomial model would be able to replicate the volatility frown.

Could use Merton’s mixed jump-diffusion model.


Adds jump to the stock price process.
dS = (r-q-λk)Sdt + σSdz + Sdp
Where λ is average number of jumps per year.
k is average jump size.
dp is the Poisson process generating jumps.

Constant Elasticity of Variance (CEV) model might be used.


Assumes stock price process dS = (r-q)Sdt + σSαdz
By changing α different volatility smiles can be addressed.
10.
• Cannot be relied on solely for ALM purposes
• Assumes constant risk and return for the assets (or static measure)
• SR is a Single factor model and does not reflect multi factor aspect of ALM

c) RACS(t) = E(t)[S(t+1) – S(t)(1+Rf)] / s(t)[S(t+1)]


Asset 1
Surplus for portfolio with 100% Asset 1: 10.0, 15.0, 21.0, 18.4, 15.9, 17.2
Return (Surplus 1): 50.0%, 40.0%, -12.4%, -13.6%, 8.2%
Excess Return = (St+1 – St(1+Rf))/St = 46%, 36%, -16.4%, -17.6%, 4.2%
Mean excess return (1): 10.44%
Standard Deviation (1): 26.31%
RACS(1) = mean excess return / std dev = 39.68%

Asset 2
Surplus for portfolio with 100% Asset 2: 10.0, 13.0, 13.4, 11.8, 12.6, 13.7
Return (Surplus 2): 30.0%, 3.1%, -11.9%, 6.8%, 8.7%
Excess Return = 26.0%, -0.9%, -15.9%, 2.8%, 4.7%
Mean excess return (2): 3.33%
Std Dev (2): 13.46%
RACS(2) = 24.74%
RACS(1) > RACS(2) so we really should choose asset 1 to support the
liability

d) RACS may be more effective than Sharpe Ratio due to the following reasons:
• Extends Sharpe Ratio to measure performance relative to liabilities, rather
than cash (risk free rate)
• Formula is based on dollar return on surplus in excess of risk-free rate over
risk in the surplus return
• Will provide better investment decision in ALM framework and with respect
to surplus
• For uncertain liabilities, the lowest surplus volatility strategy dedicates a bond
portfolio to match best estimate liabilities
• RACS evaluates other strategies relative to this minimum volatility strategy
23.
Learning Objectives:
5c) Define and apply the concepts of martingale, market price of risk and measures in
single and multiple state variable contexts.

This question verifies the knowledge of different concepts associated with martingales.
The candidate is also asked to apply these concepts in the case of a simple change in
numeraire.

Solution:
(a)(i) A martingale is a zero-drift, stochastic process, can be expressed as dθ = σ dz, where z is
a Wiener process.
The variable θ could depend on θ itself and other stochastic variables.
Its expected value at any future time = its value today. i.e., E(θT) = θ0

(a)(ii) numeraire refers to the security price that is used as the unit to measure another security
price typically with the same source of uncertainty.

(a)(iii) Define Ф = f / g where f, g are stochastic processes depending on the same source of
uncertainty. The equivalent martingale measure says that, when there is no arbitrage
opportunity.
Ф is a martingale for some choice of market price of risk. Further, if the choice of market
price of risk is the volatility of g, then the ratio of f / g is a martingale for all securities f.

(a)(iv) forward risk neutral refers to a world where the market price of risk is the volatility of the
numeraire, with respect to g.

(b) In the forward risk-neutral world with respect to g, the market price of risk is set to the
volatility of g and the process f /g simplifies to a martingale process, so the expected
growth rate = 0.
D (f / g) = (σf – σg )(f / g) dz, so the volatility of f / g is (σf - σg )(f / g)
Questions 1 – 5 pertain to the Case Study

3. (6 points) LifeCo has revised downward the effective duration of the accumulation
annuity block of business from 4.7 years to 4.1 years. You are responsible for reviewing
the portfolio of assets backing the block of business (Appendix A to the Case Study).

(a) (1 point) Demonstrate that the assets backing the accumulation annuity block of
business do not comply with LifeCo’s ALM guidelines after the revision of
effective liability duration.

(b) (4 points) Analyze the characteristics of each asset class in the portfolio that
should be considered in rebalancing the portfolio to be in compliance with the
ALM guidelines.

(c) (1 point) Recommend changes to the portfolio to bring it in compliance with the
ALM Guidelines, given that the yield curve is expected to shift upward and the
accumulation annuity block of business is coming out of its surrender charge
period.

EXAM APMV: Spring 2008 -3- GO ON TO NEXT PAGE


Advanced Portfolio Management
Morning Session
Questions 1 – 5 pertain to the Case Study

5. (6 points) LifeCo’s ALM Committee decided to fully hedge the guarantee risk in the
Equity Linked GIC portfolio effective Dec. 31, 2000. You are to prepare a static hedge
proposal using only options.

S&P Price Index, Dec. 31, 2000 1378


S&P Total Return Index, Dec. 31, 2000 1590
Interest (annual) 12%
S&P Index dividend yield 3%
Volatility (annual)
S&P Price Index = 18%
S&P Total Return Index = 18%

Assume that the Equity Linked GIC asset portfolio has tracked the S&P Total Return
Index performance very well since issue. Options with the following parameters are
available in the over-the-counter market:

S&P Total Return


Underlying
S&P Price Index
Put
Option
Call
American
Exercise
European

(a) Determine option(s) for LifeCo’s Equity Linked GIC portfolio; specify type of
underlying, option, and exercise.

(b) Determine the term, strike rate, and notional amount of the option(s)
recommended in (a).

(c) Calculate the premium of the option(s) recommended in (a) using the information
given.

(d) Demonstrate how the option(s) purchased will hedge the risk if the S&P Price
Index and Total Return Index drop to 1170 and 1350, respectively, ignoring
premium paid for the option(s) in (c).

EXAM APMV: Spring 2008 -5- GO ON TO NEXT PAGE


Advanced Portfolio Management
Morning Session
12. (5 points) The stock price of ethanol producer MPG Inc. depends on three variables: the
price of gasoline, the price of corn, and the performance of the Russell 2000 index. You
are given the following information:

Risk-free Rate r f 6%
Gasoline Forward: Expected Return 9%
Volatility 15%
Corn: Market Price of Risk 0.1
Volatility 25%
Russell 2000 stock index: Market Price of Risk -0.05
Volatility 30%

(a) Calculate the market price of risk for gasoline.

(b) Calculate the expected return on MPG Inc. stock.

(c) Define a martingale, and show its formula form.

The dollar money market account, g, follows process dg = rg dt , where r is the


instantaneous risk-free rate. In a no-arbitrage world with g as the numeraire, the price of
any traded derivative, f, depends on the same source of uncertainty as the dollar money
market account, g.

(d) Determine:

(i) the market price of risk of this world to make the ratio of f / g a
martingale.
(ii) whether this is a risk-neutral world and provide the reason for it.

Suppose that x follows the Ito process:

dx = a ( x, t ) dt + b ( x, t ) dZ

If G is a function of x and t, then Ito’s Lemma shows that G follows the process:

⎛ ∂G ∂G 1 ∂ 2G 2 ⎞ ∂G
dG = ⎜ a+ + b ⎟ dt + bdZ
⎝ ∂x ∂t 2 ∂x ∂x
2

(e) Verify that the ratio f / g is a martingale.

EXAM APMV: Spring 2008 -2- GO ON TO NEXT PAGE


Advanced Portfolio Management
Afternoon Session
13. (4 points) The yield of a stock life insurance company’s surplus portfolio has decreased
and the company’s portfolio management is considering adding hedge funds and/or
private equities. The existing asset portfolio is comprised of 20% public equities and
80% bonds, consistent with the investment policy, and is earning 5%. The company
requires a target yield of 5.25%.

(a) Outline the pros and cons to management of investing in

(i) private equities and


(ii) hedge funds.

Current Hedge Hedge Private Private


Portfolio Fund W Fund X Equity Y Equity Z
Implied premium --- 100bp 205bp 50bp 75bp
to current portfolio
Volatility 10% 10% 20% 15% 15%

(b) Analyze the above four investment options with respect to whether the investment
will meet or exceed the company’s target on a risk adjusted basis.

14. (6 points) A pool of mortgage loans backing a particular mortgage backed security has
the following information available at the end of the month, assuming the scheduled
balance resets at the beginning of each month.

Scheduled Balance $250,000,000


Actual Balance $245,000,000
Age (months) 32

(a) Calculate the Single Monthly Mortality (SMM), Conditional Prepayment Rate
(CPR) and Public Securities Association (PSA) model of the pool based on the
provided information.

b) Identify and describe the key factors that impact the prepayment behavior of
Agency mortgage backed securities.

c) Describe how the prepayment profile of subprime mortgage collateral differs from
conforming agency mortgages and identify the key underlying reasons that drive
the differences.

EXAM APMV: Spring 2008 -3- GO ON TO NEXT PAGE


Advanced Portfolio Management
Afternoon Session
16. (6 points) You own a portfolio of corporate bonds. You are interested in hedging the
value of your portfolio against changes in interest rates and credit spreads.

Given the following information:

Security DVBP Conversion Factor for CTD* Price


Portfolio 100,000 n/a 10,000,000
Treasury Future 80.5 1.20 95-05

*CTD = Cheapest To Deliver

(a) Determine the number of Treasury futures contracts needed to implement a hedge
against a small parallel shift in the yield curve.

(b) Calculate the impact on the value of the hedged portfolio determined in (a),
assuming that there is a 10 basis point upward shift in the yield curve.

(c) Identify the risk related to the changing credit spreads and describe approaches
that could be used to mitigate that risk.

(d) Identify other hedging instruments that may be used to manage interest rate risk.

EXAM APMV: Spring 2008 -5- GO ON TO NEXT PAGE


Advanced Portfolio Management
Afternoon Session
18. (6 points) A 45 year old client has accumulated an investment portfolio of $1 million
over his lifetime. His portfolio returns to date have averaged in excess of 10% per year
due to his 100% concentration in equities. He expects to retire at age 65 and expects his
employment income to be sufficient to support him and his spouse until then. At
retirement, he will sell his business and use the proceeds to provide for all his retirement
income. He has no other sources of income.

(a) Create an outline of an Investment Policy Statement (IPS) for your client and
justify your recommendation. Assume that your client has no other dependents.

(b) Revise the IPS given the following additional information: your client has an
elderly mother, who has recently fallen ill, and may have large medical care bills.
Assume your client will be responsible for paying for his mother’s future medical
care.

(c) Revise the IPS given that the client’s mother has been institutionalized and has
fixed monthly long-term care costs. Recommend revisions to the IPS, for each of
the following risk tolerances:

(i) conservative (with minimal investment risk)


(ii) moderately aggressive (with some investment risk, but also some
protection of capital)

EXAM APMV: Spring 2008 -7- GO ON TO NEXT PAGE


Advanced Portfolio Management
Afternoon Session
3.
Learning Objective:

4 – c. Describe how liability requirements affect the selection of an investment


strategy or the selection of an optimal portfolio

This question addresses management of interest rate risk in a portfolio supporting


life insurance liabilities. It draws on the Case Study and material in
Fabozzi Ch. 12, 13, 23, 24, and 27.

Solution:

(a)
ƒ The ALM guideline states: The dollar duration of assets less the dollar
duration of liabilities must be < 30% of the Book Value of Assets.
ƒ DD (Liabilities) = 1575 × 4.1 = 6458 million
ƒ DD (Assets) = 1545 × 4.7 = 7257 million
ƒ DD (Assets) − DD (Liabilities) = 801 million
ƒ 0.3× Book Value Assets = 450 million
ƒ 801 million is greater than 450 million; does not comply; there is a violation

(b)
ƒ Government bonds: highly liquid
ƒ Public corporate (investment grade): Liquid. Could be callable. Drop in
interest rates would lead to reinvestment risk. Should shift to shorter
maturities.
ƒ Private corporate (investment grade): Less liquid. Increase in interest rates
could force liquidation at loss. Should reduce exposure and shift to
shorter maturities.
ƒ Public corporate (below investment grade): Less liquidity
ƒ Pass-Throughs: Often agency backed so low credit risk. Prepayment risk
depending on tranche.
ƒ Cash and short-term: Highly liquidity. Minimal credit risk. Increase
allocation would be wise.
ƒ Real estate: Illiquid
ƒ Equity: less than 1% weight

(c)
ƒ Shift allocations of government and corporate bonds to shorter maturities to
bring down duration of asset portfolio.
ƒ Reduce allocations to private corporate bonds to reduce liquidity risk
ƒ Increase allocation to cash and short term to avoid possibility of forced
liquidation in a rising interest rate environment.
ƒ Need liquid assets to prepare for high surrenders due to expiration of
surrender fee and high market interest rate versus crediting rate.

APMV Illustrative Solutions -5- Spring 2008


5.
Learning Objective:

2 – a. Demonstrate mastery of option pricing techniques and theory for equity,


interest rate, and credit derivatives

This question asks the candidates to apply basic stock option principles to hedge a
simple product. The material is covered in Hull Ch. 14.

Solution:

(a)
Equity Linked GICs offer the return of principal after 5 years plus 75% of the
average percentage increase in S&P 500 TR index over that period. Since asset
portfolio tracks S&P TR pretty well, we could assume it is equivalent to invest in
S&P 500 TR. So to meet the guarantees we need to make sure initial principal is
protected, that means buying European put option on S&P 500 TR index.

The strike price = 1590 / 1.06 = 1500

Term: 4.5 years

(b)
The notional of the option is 55M ×0.75 = 41.25 M

(c)
Using Black-Scholes formula to price the put option
S0 = 1590 K = 1500 r = 12% vol = 18% T = 4.5 q = 0
p = 1500 × exp(−0.12 × 4.5) N (− d 2 ) − 1590 N (− d 1 )
d 1 = 1.9486 d 2 = 1.5668
p = 11.16
The number of put contracts 41.25M / 1590 = 25,943.4
Total cost = 11.16 × 25,943.4 = 289,506

(d)
If index drops from 1590 to 1500, the return of the portfolio
= (1590 − 1500) /1590 = −5.66%
The principal of the portfolio is 55M ×(1 − 5.66% × 0.75) = 52, 665, 094
If the S&P 500 TR index drops to 1350, the payoff of put contracts is
25,943.4 × (1500 − 1350) = 3,891,509, the portfolio will worth
55M ×(1 − ((1590 − 1350) /1590) × 0.75) = 48, 773,585
The total portfolio will worth 3,891,509 + 48, 773,585 = 52, 665, 094
The principal is protected

APMV Illustrative Solutions -9- Spring 2008


12.
Learning Objective:

1 – c. Define and apply the concepts of martingale, market price of risk, and
measures in single and multiple state variable contexts

These concepts underlie all interest rate derivative mathematics. The material
draws from Hull Ch. 25.

Solution:

(a)
Market price of risk λ =
(μ − r)
σ
Therefore, market price or risk for gasoline =
( 0.09 − 0.06 ) = 0.2
0.15

(b)
μ − r = ∑ λ (i ) σ (i )
Expected return on MPG stock = rstock − rf
= 0.2 × 0.15 + 0.1× 0.25 − 0.05 × 0.3
= 0.04

(c)
Martingale is a zero drift stochastic process
Has the form dθ = σ dZ
dZ is a Wiener process

(d)
i. Martingale is a zero drift stochastic process
ii. This is a risk neutral world since the market price of risk is = 0

APMV Illustrative Solutions - 22 - Spring 2008


12. continued

(e)
Use σ g as the numeraire

dg = ( r + σ f σ g ) f dt + f σ f dz
dg = ( r + σ g 2 ) g dt + g σ g dz

Using Ito’s Lemma on ln ( f ) and ln ( g )

⎡ 1 ⎤
d ln ( f ) = ⎢( r + σ f σ g ) − σ f 2 ⎥ dt − σ f dz
⎣ 2 ⎦
⎡ ⎤
d ln ( g ) = ⎢( r + σ g 2 ) − og 2 ⎥ dt − σ g dz
1
⎣ 2 ⎦

Subtracting, we get d ln ( f / g ) = (σ f − σ g ) dt − (σ f − σ g ) dz
2

Apply Ito’s Lemma again,

d ( f g ) = (σ f − σ g ) f g dz

So, f g is a martingale.

APMV Illustrative Solutions - 23 - Spring 2008


14.
Learning Objective:

4 – d. Describe and compare specialized financial instruments that can be used in


the construction of an asset portfolio supporting financial institutions and
pension plan liabilities

This question asks the candidates to calculate the Single Monthly Mortality
(SMM), Conditional Prepayment Rate (CPR) and Public Securities
Association (PSA) model of a pool of mortgage loans, describe the key
factors that impact the prepayment behavior and describe how the
prepayment profile of subprime mortgage collateral differs from
conforming agency mortgages. The material is from HFIS, Ch. 23 and 26.

Solution:

(a)
⎡ ( scheduled balance − actual balance ) ⎤
SMM = 100 × ⎢ ⎥
⎣ ( scheduled balance ) ⎦
⎡ ( 250, 000, 000 − 245, 000, 000 ) ⎤
= 100 × ⎢ ⎥
⎣ 250, 000, 000 ⎦
= 2%

⎡ ⎛ ⎛ SMM ⎞ ⎞12 ⎤
CPR = 100 × ⎢1 − ⎜1 − ⎜ ⎟⎟ ⎥
⎣⎢ ⎝ ⎝ 100 ⎠ ⎠ ⎦⎥
= 100 × ⎡1 − (1 − ( 0.02 ) ) ⎤
12

⎣ ⎦
= 21.52%

⎡ CPR ⎤
PSA = 100 × ⎢ ⎥
⎣⎢ ( min ( age,30 ) × 0.2 ) ⎦⎥
⎡ 0.2152 ⎤
= 100 × ⎢ ⎥
⎣ ( 30 × 0.2 ) ⎦
= 3.69

APMV Illustrative Solutions - 25 - Spring 2008


14. continued

(b)
The key factors that impact the prepayment behavior of Agency Mortgage Backed
Securities:
• Current interest rate level – a decrease in interest rate creates an
incentive for homeowners to refinance and this increases payment
level; conversely, an increase in interest rates will slow down
prepayment levels
• Time of the year – relocations are more frequent during
Spring/Summer
• Economic condition – affects level of relocation
• Slow of the yield curve – if the yield curve is steep, homeowners are
encouraged to borrow via ARMs and continue to refinance to take
advantage of short-term interest rates
• Burnout – refinance opportunity with a loan pool declines over time
regardless of whether rates continue to decline
• Seasoning – seasoned mortgage homeowners are more likely to
relocate and make additional partial payments

(c)
Prepayment profile of subprime mortgage collateral differs from conforming
agency mortgages
• There is generally less refinancing in subprime mortgages even if
current market rates drop below current loan rates as subprime
mortgage holders may not have sufficient good credit to get a new
loan and take advantage of rate opportunities
• More restrictions for subprime mortgage holders because they lack
good credit history to pursue alternatives
• Because of these conditions, the prepayment profile of subprime
mortgages has better convexity profile than confirming agency
mortgages
• Subprime mortgages season quicker and reach a stable state faster in
relation to conforming mortgages

APMV Illustrative Solutions - 26 - Spring 2008


16.
Learning Objectives:

3 – f. Recommend a credit risk management strategy for a given situation


4 – g. Evaluate complex situations associated with the presence of embedded
options, hedging strategies, accounting considerations, taxation and capital
requirements under a range of economic environments
4 – i. Recommend an investment strategy for a given situation, including portfolio
policy and objectives, asset selection criteria, capital market expectations,
and risk management strategy

This question explores fixed income hedging of corporate spreads and the
underlying Treasury yield curve. It draws on material in Babbel &
Fabozzi Ch. 8 and 19, and Crouhy Ch. 12.

Solution:

(a)
DVBP Portfolio
# of contracts =
DVBP Futures × Conversion Factor
100, 000
=
80.5 × 1.20
= 1, 035

Short 1,035 contracts.

(b)
After 10 bps upward shift,

Change in Portfolio Value = −DVBP × 10 = −$1, 000, 000


Portfolio Value decreases by $1,000,000

Changes in Futures Position = 1, 035 × 80.5 ×1.20 ×10 = $999,810


Futures position increases by $999,810

Change in value of Hedged Portfolio = −$1, 000, 000 + $999,810 = −$190

APMV Illustrative Solutions - 29 - Spring 2008


16. continued

(c)
The risk related to the portfolio is widening credit spreads; which implies a
reduction in the value of the bonds.

To mitigate the risk, can purchase


• Credit spread options
• Credit default swaps
• Credit spread forwards
• Credit linked notes
• Total return swaps

Use Credit Linked Notes

Buying put options on sector index

Shorting a basket of stocks in the sector

(d)
Treasuries
Advantages
ƒ Liquid, default free

Disadvantages
ƒ Can not hedge spread risk, on balance sheet, some issues may
be in short supply (expensive to borrow)

Interest Rate Swaps


Advantages
ƒ Off balance sheet, no supply problems, good to hedge spread
risk

Disadvantages
• Less liquid than futures, liquidated through counterparty (rather
than traded), counterparty risk

Interest rate caps / floors

APMV Illustrative Solutions - 30 - Spring 2008


18.
Learning Objective:

4 – i. Evaluate the need to revise an investor’s investment policy and recommend


revisions over a lifetime

This question asks the candidates to create an investment policy statement for a
client, amend it for a real life situation and then discuss solutions. The
material draws from Magnin Ch.11, Babbel & Fabozzi, Ch. 26 and HFIS
Ch. 48.

Solution:

(a)
• Liquidity: must provide some in case of unexpected situations, however not
much is needed due to current salary
• Risk tolerance: current has 100% in equities, high risk tolerance. Plans to use
proceeds of business sale to buy retirement income so not counting on
investment to fund retirement, therefore high risk tolerance
• Time horizon: plans to retire in 20 years, long time horizon allows for more
risk taking
• Objectives: enjoying 10% return

100% in equities is risk, however, doesn’t rely on this income for the future

(b)
• Liquidity: must increase liquidity due to possible cost. Equities are fairly
liquid, however, $1M to liquidate once may be too much, may start
investing in liquid GIC.
• Risk tolerance: decrease significantly, must ensure sudden drops in value of
equities may deplete portfolio when decide to liquidate
• Time horizon: may need cushion soon. Can no longer focus in long-term
return
• Objectives: may need safe and steady return

(c)
(i)
Conservative
• Portion of portfolio switches to coupons payable bonds that cover
monthly care of mother
• Can invest majority in fixed income investment and leave some in
equity for future appreciation and possible future long-term care
cost inflation

APMV Illustrative Solutions - 33 - Spring 2008


18. (c) continued

(ii)
Moderate
• Match future payments to long-term care exactly with coupons
payable bonds and leave the rest in equity

APMV Illustrative Solutions - 34 - Spring 2008


**BEGINNING OF EXAMINATION**
MORNING SESSION

Questions 1 – 4 pertain to the case study

1. (6 points) LifeCo management is concerned about the dollar duration mismatch in the
traditional line of business. It has been suggested that the company increase its exposure
to non-agency Collateralized Mortgage Obligations (CMO) as a means of closing the
duration gap. You have been assigned to study this strategy.

(a) Describe non-agency CMOs.

(b) Compare agency and non-agency CMOs.

(c) Assess the appropriateness of this new investment strategy.

(d) Determine whether this strategy complies with LifeCo’s investment policy for the
traditional line.

EXAM APMV: Spring 2009 -1- GO ON TO NEXT PAGE


Advanced Portfolio Management
Morning Session
Questions 1 – 4 pertain to the case study

2. (8 points) As the Chief Investment Officer for LifeCo, you are looking to implement
benchmarks for the fixed income portfolios.

(a) List and describe the basic steps in developing quantitative portfolio management
techniques relative to an index.

In order to better analyze and manage the portfolio risk and performance of LifeCo’s
fixed income assets backing the $1.5 billion of accumulation annuities, you are reviewing
the following XYZ Index:

XYZ Index Composition % in Index Effective Duration


Investment Grade AA 15% 6.0
Investment Grade A 32% 7.0
Below Investment Grade BB 5% 6.0
Below Investment Grade B 3% 4.0
Mortgage Products (including structured) 25% 7.0
Equity 20% 6.0
Total 100% 6.5

(b) Compare the XYZ Index to the ALM guidelines in Section V of LifeCo’s ALM
Policy Statement.

(c) Evaluate the appropriateness of the XYZ index, given LifeCo's investment
constraints:

(i) With respect to the current asset composition.


(ii) As a benchmark.

(d) Describe the complexities arising as a result of the fact that, in general, fixed
income benchmarks are capitalization weighted and all-inclusive.

EXAM APMV: Spring 2009 -2- GO ON TO NEXT PAGE


Advanced Portfolio Management
Morning Session
Questions 1 – 4 pertain to the case study

4. (8 points) LifeCo is considering entering into the credit default swap (CDS) below to
generate additional income and to enhance its risk management. LifeCo management
does not want to have more than 19% of the Non-Traditional Life segment invested in
below investment grade bonds. The CDS protects the purchaser against a rating
downgrade to below investment grade.

Equivalent Annual Premium 130bps


Notional $10 million
Asset-Backed Securities Residential Mortgages

(a) List and describe the key components of the derivative policy for this initiative.

(b) Describe how LifeCo could use this CDS to increase the yield on assets.

(c) Calculate the amount of the CDS needed to increase the segment yield to 7.25%
to support crediting rates in its Non-Traditional Life segment.

(d) Describe the losses that the LifeCo’s Non-Traditional Life segment may realize
from the arrangement described in (c) due to recent downgrades in the mortgage
sector.

(e) Explain whether this strategy is consistent with LifeCo management’s fiduciary
obligations.

EXAM APMV: Spring 2009 -4- GO ON TO NEXT PAGE


Advanced Portfolio Management
Morning Session
6. (4 points)

(a) Describe the risks embedded in Collateralized Mortgage Obligations.

(b) Describe the special considerations of mortgage passthroughs with coupons above
current market rates.

(c) Recommend approaches to hedge each risk identified in (a).

7. (5 points) A portfolio has the following credit rating profile:

Rating Percentage
Aaa 5%
Aa 10%
A 30%
Baa 50%
Ba 5%

Given the following 12 month credit transition frequency matrix:

12 Month Transition (percent)


Initial Rating Aaa Aa A Baa Ba B Caa-C Default
Aaa 85 10 5
Aa 80 10 5 5
A 75 10 10 5
Baa 75 10 10 5
Ba 5 65 15 10 5
B 5 60 20 15
Caa-C 10 65 25

(a) Calculate the expected credit rating profile of the portfolio at the end of 12
months.

(b) Describe how a credit default swap (CDS) works.

(c) Assess how CDS can be used to hedge credit risk in the portfolio.

EXAM APMV: Spring 2009 -6- GO ON TO NEXT PAGE


Advanced Portfolio Management
Morning Session
19. (6 points)

(a) Compare and contrast growth and value styles of equity investing.

(b) Compare and contrast returns-based with holdings-based analysis of equity


portfolios.

(c) Describe the limitations and considerations in returns-based and holdings-based


analyses.

(d) Interpret the results of the analyses below to assess the investment style of an
equity portfolio:

According to a holdings-based style analysis as of June 6/30/20XX:

Portfolio Market Benchmark


Number of stocks 30 750
Dividend yield 2.5% 2.1%
P/E 18 20
P/B 1.8 2
EPS growth (5-year projected) 15% 12%
Sectors
Finance 15% 10%
Health Care 20% 20%
Information Technology 25% 30%
Materials 15% 10%
Telecommunication 10% 10%
Industrials 5% 5%
Utilities 10% 15%

According to a returns-based style analysis for 36 months ending 6/30/20XX.

Effective style
large-cap growth style 20%
small-cap growth style 52%
mid-cap value style 15%
mid-cap growth style 13%

style fit (R2) 97.8%


annualized active return 0.3%
annualized tracking risk 6%

EXAM APMV: Spring 2009 -9- GO ON TO NEXT PAGE


Advanced Portfolio Management
Afternoon Session
APMV Complete Illustrative Solutions
Spring 2009

1.
Learning Objectives:

2 – a. Describe and compare specialized financial instruments that can be used in


the construction of an asset portfolio supporting financial institutions and
pension plan liabilities

Solution:

(a)
• Collateral of non-agency MBS usually generated from loans that are not
conforming to agencies (GSEs), loan size usually > $360,000 (loan limits,
documentation)
• Underlying collateral is residential mortgages
• Collateral that support non-agency CMOs include:
• Home equity loan (include other home loans)
• Hybrid adjustable-rate mortgages (ARMs) (usually have fixed rate in
the first year, then mortgage rate become adjustable – Have ‘teaser
rate’ in early years
• Alt-A mortgage (borrower’s income is not verified – such as when
borrower is self employed)
• Jumbo loan (loan size is not conforming to agencies)
• Subprime mortgages
• Compensating interest (borrowers can repay the mortgages during any
day of the month, but agencies guarantee and pay interest as if all
prepayments occur at the end of the month. So investors get a full
month’s worth of interest)
• Weighted-average coupon dispersion (the standard pooling for whole
loan is looser, so higher dispersion of coupon mortgage and maturity)
• Clean-up call provision (issuers usually have the option to pay-off
outstanding debt to avoid high fixed servicing cost. Investors need to
be aware of this as it will shorten the life of back-end tranches)
• Usually have credit enhancements to mitigate its credit risk:
• External (corporate guarantee, letters of credit, bond insurance, pool
insurance)
• Internal (reserve fund, excess spread account, senior/subordinated structure)
• Subordinate classes are ‘shifting’ interest to cover defaults move money to
senior tranches earlier

APMV Complete Solutions -1- Spring 2009


1. continued

(b)
• Agency CMOs usually backed by government-sponsored enterprise (GSEs)
such as Freddie Mac, Ginnie-Mae, Fannie Mae
• Different from agency CMOs, non-agency is not backed by full faith of US
government
• Agency CMO uses a general prepayment model while non-agency CMO uses
a specialized prepayment model to capture the unique characteristics of
prepayment of the collateral
• The collateral is usually non-conforming loans

(c)
The effective duration of liabilities is 13 but effective duration of the assets is 7.5.
Dollar duration is the effective duration times the book value. Therefore, the
dollar duration of the liabilities is much higher than the dollar duration of the
assets. Assuming the non-agency CMOs have the same effective duration as the
CMOs currently backing the Traditional line of business, increasing the exposure
to non-agency CMOs would lower the dollar duration of the assets since the
effective duration of the CMOs is 4.5.

This new investment is not appropriate since it lowers the dollar duration of the
assets. Instead of lowering the dollar duration of the assets, LifeCo should
increase the dollar duration of the assets.

A better strategy would be to increase the exposure to Government securities or


investment Grade public corporate bonds.
• Agency have less prepayment risk and have virtually no credit risk
• Non-agency CMOs have more prepayment risk and credit risk
• Investing in non-agency CMOs is not appropriate for the traditional
line. First, CMOs are subject to prepayment risk so duration is
uncertain. Non-agency CMOs are also subject to high credit risk due
to subprime mortgages, so it poses high risks for not meeting the
obligations
• Life Co would also like to reduce credit risk as the current portfolio
already has higher low grade bonds than the target, but investing in
non-agency CMOs will add credit risk
• Risk: prepayment risk, extension risk, default risk
• Main risk is prepayment risk. As interest rates decrease, many
prepayments may occur to refinance
• CMOs are not a good idea – will result in interest rate risk,
reinvestment risk, and if non-agency, have credit risk
• If the CMOs prepay, it will happen when rates are low and
reinvestment will be at low rates causing spread compress
• Non-agency CMOs have servicer bankruptcy risk

APMV Complete Solutions -2- Spring 2009


1. continued

(d)
• From the Assets and Liabilities Income sheet on p. 11, CMOs have duration
of 4.5. This will not decrease the duration Gap, unless the new CMOs have a
higher duration
• They also have a negative convexity
• The strategy would not comply with the policy as they are not listed. They are
not a good source of guaranteed return.
• Traditional life has mismatching of D (A) – D (L). D (A) is 7.6 which is
shorter than D (L) = 13.0. However non-agency CMOs have prepayment risk.
Once interest rates fall, non-agency CMO may refinance depending on credit
quality, loan rate, economy situation, balance size, seasonal effect.
• Usually non-agency CMO has low credit quality. Therefore, they have high
loan rate. If their incentive to refinance has been increased and they have
alternatives to refinance, prepayment risk exists. It reduces duration of assets
even further. Asset liability mismatch gets serious.
• Life Co’s investment policy has ALM guidelines. It has guidelines for asset
quality. Credit risk exposure should be diversified and monitored. Non-
agency CMOs have a problem on credit risk unless they have extra
enhancements.
• Also, Traditional Life has guidelines for duration and scenario tests to comply
with the guidelines. Non-agency CMOs have volatile cash flows. To sum,
non-agency CMO usually in conflict with current investment policy.
• Life Co would like to minimize the duration mismatch of traditional line.
According to the current portfolio, CMO only have a duration of 4.5, so it
cannot correct the mismatch problem unless it is invested in very long
tranches.

APMV Complete Solutions -3- Spring 2009


2.
Learning Objectives:

The three syllabus materials discuss application of benchmarks to fixed income


portfolios. This question would have three parts. First, would ask for a general
description of the benchmarking process as it is applied to fixed income
portfolios. Second, would be to analyze and recommend a customized benchmark
for Life Co’s accumulation annuity portfolio. Third, would be to discuss the
issues/challenges in selecting fixed-income benchmarks.

Solution:

(a)
• Formulate an investment policy in terms of limits to
• Diversification and liquidity requirements
• Duration targets

• Select a benchmark
• A standard benchmark
• Or a customized benchmark

• Analyze a portfolio relative to the benchmark along multiple risk dimensions


to identify exposures

• Quantify risk exposures based on historical volatilities and correlations

• Compare projected portfolio performance versus its benchmark under a set of


scenarios

• Establish a methodology for replicating the benchmark return in absence of a


market view and creating proxy portfolios

• Optimize the portfolio subject to constraints on deviation from the benchmark

• Attribute achieved return to risk exposures taken

(b)
Dollar Duration of Index Assets 1,500 × 6.5 = 9,765
Dollar Duration of Liabilities 1,500 × 4.7 = 7,050
Dollar Duration of Index Assets less Liabilities 2,715

Required DD of 1,500 × 0.3 450

Dollar duration of XYZ Index is greater than the ALM guidelines

APMV Complete Solutions -4- Spring 2009


2. continued

(c)
(i)
Current Assets Book Value % of Total
Government 59 4%
Total Investment Grade Corporate 728 49%
Total Below Investment Grade Corporate 258 17%
Mortgage Products 369 25%
Equity 19 1%
Cash 30 2%
Other 38 2%
Total 1,500 100%

• Total Investment Grade for XYZ Index is 47% (32% + 15%) versus
49% for current assets, which fits well
• Total Below Investment Grade for XYZ Index is 8% (5% + 3%)
versus 17% for current assets, which does not match well
• Total Mortgage Products for XYZ Index is 25% versus 25% for
current assets, which fits well
• Total Equity for XYZ Index is 20% versus 1% for current assets,
which does not match well
• XYZ Index does not have any allocation to Government and Cash
positions which does not match well

(ii)
• An appropriate benchmark should match the desired or required strategic
allocation of portfolio assets so that the portfolio manager is able to “buy the
benchmark” when and if he so desires
• The standard weighting for XYZ Index is not appropriate for the
current assets
• Goal should be to keep the benchmark as broad based and well-diversified as
possible while still meeting all the requirements of the investment policy
• XYX Index is broad based and well-diversified
• Even with customized weights, the objectivity of the benchmark should be
preserved in order to allow for historical analysis

(d)
Duration Problem
The duration problem is the fact that the duration of the benchmark comes
from issuer preferences and is not necessarily the duration that a given
investor should hold

“Bums” Problem
The bums problem is that the biggest debtors have the largest weights in
the benchmarks

APMV Complete Solutions -5- Spring 2009


4.
Learning Objectives:

1 – b. Identify the obligations of a fiduciary in managing investment portfolios


and show how they apply in a given situation.

Solution:

(a)
Reference: Managing your Advisor – V-C138-09 p. 22-23 – Items 1-6 as relevant

Accountability
• ALM committee review type and amount of purchase; inv dept
executes trade; finance is responsible for accounting reporting.
Permitted Uses
• Hedging only for improving risk management
• Investment risk reduction – same idea as above; could buy a Corp A
bond and protect against 1 downgrade or 2 or whatever mgmt is
comfortable with
• Replication – could be an option here since could write the CDS and
hold governments, which means you are holding a lower rated security
Type of derivatives allowed
• CDS are on approved list
Counterparty restriction
• Need to have exposure limits to names, especially important on CDS’s
Derivative Portfolio exposure limits
• Should establish a limit in the portfolio and for the company (via the
surplus segment) – should be tied to maximum tolerance for loss (say
no more than 5% of segment and/or 5-10% of surplus but this will
depend on actual level of surplus versus required) – could look at VAR
analysis
Internal Controls
• Ensure they are appropriate and consistent with other derivative
policies – key point here is monitoring external environment and
terminate program if conditions deteriorate, ensure diversification and
do not over expose to one issuer or sector
• FAS 133

APMV Complete Solutions -9- Spring 2009


4. continued

(b)
Reference: LifeCo case study – p. 12, 30, 32 Non-Trad segment

• Sell protection to a protection buyer


• LifeCo receives a fee (130 bps of notional)
• LifeCo will pay in the event the mortgage defaulted up to the notional amount
• Must specify in the contract what constitutes default
• Could invest more heavily in MBS and offset risk by purchasing CDS, payout
130 bps fee, but are covered in case of a default, this effectively rises the
credit standing of the MBS

(c)
Reference: LifeCo cast study – p. 12, 30, 32 Non-Trad segment

• Current yield = 7.17%


• Current exposure = 400 million
• Target yield = 7.25%
• Need yield ( 7.25% − 7.17% ) × 400 = 0.32M
0.32
• Need to sell = = 24.6 notional amount of CDS
1.3%

(d)
Reference: Maximum loss determined by reviewing Term Sheet from Case Study
p. 32

• If recent downgrades indicate an expected increase in defaults, then LifeCo


may need to pay out claims on its CDS contracts
• Losses are capped at 24.6 million notional
• Losses aren’t likely to reach 24.6 million because of recoveries in the mortgages
• Pay outs are commonly (Notional – Market value of mortgage) if the credit
event is triggered

(e)
Reference: Fiduciary study note FET-128-07 p. 5, 8, 10

Fiduciary obligations
• Loyal
• Make property productive
• Diversify
• Delegate appropriately
• Act in accordance with trust
• Fiduciary obligation is not to take excessive risk w/ principal
• This strategy has big downside risk
• Is consistent with LifeCo fiduciary obligations

APMV Complete Solutions - 10 - Spring 2009


6.
Learning Objectives:

5 – k. Demonstrate how to apply funding and portfolio management strategies to


control equity and interest rate risk, including key rate risks. Explain the
concepts of immunization including modern refinements and practical
limitations.

Source: Babbel & Fabozzi, Chapter 31, Pages 433 - 441

Solution:

(a)
Spread Risk
• The risk of widening and narrowing of the Option Adjusted Spread
(OAS)
• It is used to compensate for the extra risk inherent in mortgage backed
securities
• OAS is calculated by projecting cash flows using a prepayment model
• Use prepayment model that assigns an expected prepayment
implying an expected cash flow – for a given interest-rate path
• Discount these expected cash flows at U.S. Treasury rates plus
OAS to obtain their present value
• Average the present value of the cash flows across all paths
• OAS is selected such that the average price is equal to the
observed market price of the security
• OAS is very sensitive to the prepayment model structure
• Normally do not hedge this risk

(b)
Interest Rate Risk
• The interest-rate risk of a mortgage backed security corresponds to the
interest rate risk of comparable Treasury securities
• This risk can be hedged directly by selling a package of Treasury
Notes or interest-rate futures
• Duration is the normal quantification of interest rate risk
• Interest rate curves normally do not move in tandem
• Shorter term rates (2 year) normally move twice as much as longer
term rates (30 years)
• Interest rate movement could be explained by two factors or effects:
level effect and twist effect

APMV Complete Solutions - 15 - Spring 2009


6. continued

(c)
Prepayment Risk
• The risk average lives (or durations) of mortgage backed securities
vary as interest rates change due to prepayment behavior
• The average lives extend as interest rates rise and shorten as interest
rates fall
• The percentage decline in price declines faster as interest rate rise :
negative convexity
• Dynamically hedge using package of Treasury notes or interest-rate
futures or options
• By both dynamically hedging and buying options will entail some
form of premium. Buying futures – after prices have risen and selling
futures after prices fall (buy high, sell low)

(d)
Volatility Risk
• The embedded option of the homeowner’s prepayment tends to be
more valuable when future interest-rate volatility is expected to be
high than when it is expected to be low
• OAS spreads tend to widen when expected volatility increases and
narrow when expected volatility declines
• If one believes the implied volatility is overstated as compared to the
expected future volatility, then it will be more economical to do
dynamic hedge, otherwise the purchase of options will be more
economical

(e)
Model Risk
• The risk that the prepayment model is wrong
• Models calibrated to past behavior will understate the impact of
innovation
• Cannot be hedged explicitly but could keep portfolio exposure in line
with that of the indices

APMV Complete Solutions - 16 - Spring 2009


19.
Learning Objectives:

4 – b. Assess a portfolio position against portfolio management objectives and


recommend a strategy to rebalance the portfolio

This is a recall and evaluate question asking candidates to review results of an


active equity investment analysis. The correct answers are expected to be based
on the content in Section 5 Active Equity Investing in Ch. 7 “Monitoring and
Rebalancing” of Maginn’s book. Credit is given for answers that provide correct
investment style differences, explain stated style analyses, and infer reasonably
the style of the portfolio.

Solution:

(a)
Value style
• Low P/E
• Look for stocks selling at low prices to current or normal earnings
• In industries categorized as defensive, cyclical, or out-of-favor
• Expect their P/E to rise as they recover
• Contrarian
• Look for stocks selling at low P/B
• In depressed industries
• High yield
• Look for stocks with high and stable dividend prospects

Growth style
• Focus on earnings
• Pay above-market earnings multiples for companies that have superior
growth rates
• Invest in growing industries, i.e. high sales growth, high P/Es P/Bs
• Consistent growth: long history of sales growth, high profitability,
predictable earnings
• Earnings momentum: have higher EPS growth, but such growth is less
sustainable
• Buy stocks at a premium and count on market to continue paying it for
earnings growth

APMV Complete Solutions - 40 - Spring 2009


19. continued

(b)
Returns-based style
• Relies on portfolio return to understand the characteristics of portfolios
• Regresses portfolio returns on the return series of securities indices
• Analyzes historical returns of the portfolio
• Characterizes entire portfolio
• Facilitates comparisons of portfolios
• Aggregates effect of investment process
• Different models give consistent results
• Clear theoretical basis
• Quick and cost effective

Holding-based style
• Categorizes individual securities by their characteristics; and
• Aggregates results to understand overall style of the portfolio
• At a given point of time
• Characterizes each position
• Facilitates comparisons of individual positions
• Captures changes quicker than returns-based
• More data intensive

(c)
Returns-based
• Need to properly select indices, that are mutually exclusive
• Error in specifying indices may lead to inaccurate conclusions
• May be ineffective in characterizing current style

Holdings-based
• Need to properly select possible variables, EPS, P/E, sector ...
• May not reflect the way managers approach security selection
• Different spec may give different results

(d)
Holdings-based analysis
• Lower P/E – value
• Lower P/B – value
• Higher EPS growth – growth
• Higher dividend yield – value
• Higher weights in Finance – value
• Lower weight in utilities – growth
• Lower weightings in IT – value
• A value style portfolio

APMV Complete Solutions - 41 - Spring 2009


Questions 1 – 6 pertain to the Case Study
Each question should be answered independently.

4. (5 points) You are a newly appointed ALM actuary for LifeCo. Your first tasks are to
review the current General Account portfolios backing LifeCo’s pension business and
recommend any changes to its investment policy.

(a) (1 point) Describe typical constraints on asset sales and how they could influence
decisions about portfolio rebalancing.

(b) (2 points) Explain how each of the 4 strategies listed below can help the
company mitigate risk and create value.

(i) Keeping required capital on assets under 3%.

(ii) Minimizing duration mismatch with asset sales.

(iii) Minimizing duration mismatch without asset sales.

(iv) Minimizing liquidity and marketability risk.

(c) (2 points) Recommend changes to asset allocations for LifeCo’s Payout Annuity
Portfolio separately for each of the 4 strategies in (b).

Exam APMV – Spring 2010 -3- GO ONTO NEXT PAGE


Advanced Portfolio Management
Morning Session
Questions 1 – 6 pertain to the Case Study
Each question should be answered independently.

6. (5 points) LifeCo’s ALM Committee is concerned about the interest rate risk inherent in
the Company’s institutional pension business. The Committee is considering using
floating rate assets (floaters) as part of the ALM strategy for the floating rate liabilities in
this block of business. The two floaters under consideration have 11% caps, and are
payable in U.S. dollars.

Floater Coupon Formula Maturity Date Price


A LIBOR + 30 basis points 31/12/2013 $98
B Fed Funds + 30 basis points 31/12/2013 $102

(a) (1 point) Discuss the major risks associated with ALM applications using
floaters.

(b) (1 point) Explain how LifeCo could use floaters for ALM and the risks to be
addressed.

(c) (1 point) List the factors that could explain the differing prices of the two
floaters.

(d) (2 points) Describe 2 other strategies employing floaters that provide


opportunities to add value.

Exam APMV – Spring 2010 -5- GO ONTO NEXT PAGE


Advanced Portfolio Management
Morning Session
8. (5 points) UpNorth Company has sold various insurance products including a sizable
block of payout annuities with a cost of living adjustment. Its asset portfolio consists of
only cash, equities, commercial mortgage loans, mortgage backed securities, and
corporate bonds.

To diversify the asset portfolio and manage the inflation risk in its liabilities, you propose
making new investments in commodities and inflation-linked bonds.

You have identified an inflation-linked bond with the following characteristics:

• Maturity 3 years from now


• Annual coupon payments
• Real yield: 3%
• Principal paid at maturity will be based on changes in the consumer price
index (CPI) over the life of the bond.
• At issue: Original principal = $1,000, and CPI0 = 100.00.

You project that the CPI in the next two years will be: CPI1 = 102.00, CPI 2 = 106.00.

(a) (1.5 points) Explain how the new assets might improve asset diversification and
inflation risk management.

(b) (1.5 points) Describe the economic drivers of return for long-only commodity
indexation.

(c) (1 point) Calculate the coupon payments in Year 1 and Year 2 for the inflation-
linked bond.

(d) (1 point) Determine CPI3 assuming the inflation-linked bond held to maturity
would realize a nominal yield of 7%.

Exam APMV – Spring 2010 -8- GO ONTO NEXT PAGE


Advanced Portfolio Management
Morning Session
14. (5 points) Annuities-R-Us Insurance Company has recently entered a new market
providing immediate annuities with a nonforfeiture provision to individuals. Under the
contract, the policyholder has a right to surrender the annuity and get a lump sum. The
lump sum will be calculated as the present value of future annuity cash flows discounted
at the Treasury curve + 3%. The pricing actuary indicated that this product should be
invested in higher-quality assets with spreads to Treasuries of 1.5% on average.

An actuarial student proposed investing the assets into longer duration BB zero-coupon
bonds yielding a spread to Treasuries of 3% on average. The student’s rationale is a
steeply sloped yield curve and the potential for maximizing profits.

(a) (1 point) Define Liquidity, Market and Liquidation Risks.

(b) (2 points) Critique the actuarial student’s suggested investment strategy.

(c) (1 point) Recommend the best of the following bond investment strategies for
this product and justify your recommendation.

(i) Bullet

(ii) Barbell

(iii) Laddered Maturity

(d) (1 point) List strategic and operational considerations of liquidity risk


management for Annuities-R-Us.

Exam APMV – Spring 2010 -2- GO ONTO NEXT PAGE


Advanced Portfolio Management
Afternoon Session
22. (5 points) You are a consultant to a defined benefit pension plan. The plan’s most recent
actuarial valuation revealed the following:

Assets ($ millions) 350


Liabilities ($ millions)
Active members 100
Retired members 300

A student actuary has been asked to discuss steps in developing a strategic asset
allocation for the plan. She makes the following observations:

(i) The key decision points are:


• The bond/equity split
• The allocation to alternative asset classes

(ii) The relevant asset classes to consider are:


• Bonds
• Public equities
• Private equities
• Hedge funds

(iii) Assumptions determined based on historical data over the last 10 years
will be used to find an efficient frontier.

(iv) The allocation that maximizes the Sharpe ratio will be selected.

(a) (3.5 points) Assess each statement above separately, and propose adjustments as
necessary.

(b) (1.5 points) Recommend approaches that will improve the strategic asset
allocation process for the pension plan.

Exam APMV – Spring 2010 - 10 - GO ONTO NEXT PAGE


Advanced Portfolio Management
Afternoon Session
23. (4 points) You are currently using a static model to manage a portfolio of Agency
Mortgage Backed Securities (Agency MBS). However, you are considering upgrading to
a single-period, stochastic model.

(a) (1 point) Explain the information provided to investors by the following Agency
MBS metrics:

(i) Weighted-average coupon

(ii) Weighted-average maturity & weighted-average loan age

(b) (1 point) Describe the major drivers of prepayments for Agency MBS.

(c) (2 points) Compare and contrast features and limitations of:

(i) a static model,

(ii) a single-period, stochastic model,

(iii) and a multi-period model.

** END OF EXAMINATION **
AFTERNOON SESSION

Exam APMV – Spring 2010 - 11 - STOP


Advanced Portfolio Management
Afternoon Session
4. Learning Objectives:
(1c) Determine how a client‟s objectives, needs and constraints affect the selection of
an investment strategy or the construction of a portfolio. Considerations include:
Funding objective
Investment policy
Risk-return trade-off
Regulatory requirements
Target rating from rating agency
Risk appetite
Liquidity constraints
Capital, tax, and accounting considerations.

Sources:
V-C138-09 Managing Your Advisor

V-C13-07 Investment Case Study

CIA Educational Note: Liquidity Risk Management

Commentary on Question:
This is a recall and analysis question asking candidates to evaluate investment strategy
for LifeCo‟s pension portfolio. The correct answers are expected to be based on the
content in study notes V-C138-09 (Managing Your Advisor) and the Case Study. Credit
is given for answers that provide correct investment policy objectives and constraints,
identify constraints on the asset sales and recommend improvements to LifeCo‟s asset
portfolio consistent with the objectives provided.

Solution:
(a)
Accounting
o Consider the effect of realizing gains/losses on the relevant accounting
bases
Tax Considerations
o Consider the tax implications of realizing gains/losses
Embedded Value / Economic Value Added
o Ensure that EV/EVA is maintained when executing portfolio trades
ALM Issues
o Ensure that the portfolio is rebalanced to maintain duration match
Credited Rates
o Asset portfolio returns should support the credited rates on liabilities
Policyholder Equity Issues
o Maintain appropriate diversification/asset mix

APMV Spring 2010 Solutions Page 6


4. Continued

(b)
(i)
Maintaining adequate required capital is necessary as a buffer against
potential losses.
However, LifeCo can manage the asset portfolio efficiently in order to
reduce required capital and invest the funds in value-added projects,
thus adding to the overall profitability of the company.

(ii)
Reduce earnings & surplus sensitivity to interest rate fluctuations.
Reduce C3 – interest rate risk capital and reduce losses/volatility that
may be incurred from interest rate fluctuations.
Timing of gain/loss realization can profit LifeCo and add value to the
company.

(iii)
By matching duration without asset sales, LifeCo can avoid tax and
crediting rate consequences.

(iv) Mitigating liquidity & marketability risk can help LifeCo:


Avoid “fire-sales” due to liquidity crunch,
Prepare for contingencies such as a „run on the bank,
Determine liquidity resources available in order to meet liquidity need.

Other benefits of strategies (i) thru (iv):


Maintain LifeCo‟s credit standing from rating agencies,
Meet regulator requirements, limits, etc,
Avoid rating downgrades,
Improve LifeCo‟s reputation and attract new business/maintain consumer
base.

(c)
(i)
Current required asset capital on Payout Annuities Portfolio is
32.4/700 = 4.63%. This is over the company‟s target.
Sell any of the following capital intensive instruments:
o Equities, at 20% capital
o Below Investment Grade Private Placement Bonds at 7%
o Commercial Mortgages at 5%
o Below Investment Grade Public Bonds at 4.69%

APMV Spring 2010 Solutions Page 7


4. Continued

Replace the above with any of the following low capital intensity
instruments:
o Government Securities
o Investment Grade Bonds
o Pass-through and CMO Cash
For example, switching from Commercial Mortgages to CMOs will
reduce the current required capital asset from 4.63% to below 3%.

(ii)
Asset duration is 6.1 compared to liability duration of 7.3, a mismatch
of 1.2.
To bridge the gap between asset & liability duration, LifeCo can sell
low duration assets (eg. Short duration corporate bonds) and purchase
assets with longer duration (long duration corporate bonds).
Trades should be timed and executed by analyzing yield curve.

(iii)
LifeCo can minimize duration mismatch without asset sales by using
derivatives to lengthen portfolio duration.
For example, LifeCo can set up an interest rate swap agreement to pay
floating rates in return for long term fixed rates.

(iv)
Asset mix has 5% in Below Investment Grade Public Corporate bonds
and 6% in Below Investment Grade in Private Corporate Bonds. This
is below the Investment Policy limit set at 20% and 25% respectively.
The biggest illiquid investment is in commercial mortgages,
potentially in violation of LifeCo‟s investment policy limit asset mix.
One possible justification is that Payout Annuities are illiquid
liabilities, so relatively high allocation to illiquid Commercial
Mortgages could be warranted.
LifeCo could rebalance its portfolio to hold less of Commercial
Mortgages and more of liquid asset classes such as Government or
Public bonds.

APMV Spring 2010 Solutions Page 8


6. Learning Objectives:
(2a) Describe and compare specialized financial instruments that can be used in the
construction of an asset portfolio supporting financial institutions and pension
plan liabilities.

Source:
Fabozzi, Handbook of Fixed Income Securities, 7th Edition, 2005, Chapter 16, Floating –
Rate Securities (pp. 373-379, 382-383)

Commentary on Question:
This question combines recall of the risk characteristics and uses of floaters within an
ALM program, as well as the evaluation of the appropriateness of floaters to mitigate the
risks inherent in LifeCo‟s product portfolio.

Solution:
(a) Cap Risk
The floater‟s coupon rate likely will be capped, whereas the short-term funding
may not be.

Basis Risk
The floater‟s reference rate may not be the same as the reference rate for funding.

Price Risk
If the floater‟s risk changes for the worse, the quoted margin will no longer
compensate the investor for the security‟s risks.

(b) 1. The asset/liability management strategies can be used to manage the floating
rate funding agreements.

2. LifeCo can invest in floating rate products matching the product duration.

(c) 1. Time remaining to the next coupon reset date.

2. Changes in the market‟s required margin/credit quality.

3. Possibility of cap or floor, and whether or not the cap or floor is reached.

(d) Risk Arbitrage Strategies


For example, money managers using leverage to invest in securities that earn a
higher spread over their borrowing rate. The manager will be exposed to cap risk,
basis risk and price risk.

APMV Spring 2010 Solutions Page 11


6. Continued

Betting on Changes in the Required Margin


If the floater‟s risk does not change and the compensation demanded by the
market does not change either, the floater‟s price will be par on every coupon
reset date. If conditions change such that the required spread is greater than (or
less than) the quoted margin the floater will trade at discount (premium) to par.

Arbitrage Between Fixed and Floating Rate Markets Using Asset Swaps
An asset-based swap transaction involves the creation of synthetic security via the
purchase of an existing security and the simultaneous execution of a swap.

APMV Spring 2010 Solutions Page 12


8. Learning Objectives:
(2a) Describe and compare specialized financial instruments that can be used in the
construction of an asset portfolio supporting financial institutions and pension
plan liabilities.

Sources:
V-C146-09, Greer, R., “The Role of Commodities in Investment Portfolios,”

CFA Conference proceedings Quarterly December 2007, pp. 35-46. Vol 24 #4

Ch. 15, Inflation-Linked Bonds, in Handbook of Fixed Income Securities, 7th Edition,
2005

Commentary on Question:
This question asks candidates to explain how non-standard assets can be used to
complement an existing portfolio, as well as to discuss the economic drivers of a specific
type of bond. Finally, it requires candidates to demonstrate knowledge of inflation-linked
bonds through two calculations.

Solution:
(a) Commodities have historically been shown to have negative correlation with both
equities and fixed income and, therefore, provide a good form of diversification.
They also have been shown to be positively correlated with the inflation rate.

Inflation-linked bonds provide good diversification with traditional financial


assets due to a low correlation with those assets. Cash flows are linked to the
inflation rate because the principal amount grows with inflation. This provides
good inflation protection.

(b) The economic drivers of return for long-only commodity indexation are:
T-Bill return – represents the return earned on the collateral
Risk premium – the assumption of price risk of commodities
Rebalancing – reflects the fact that commodities are not highly correlated with
each other
Convenience yield – evident when there is low inventory relative to market
demand
Expectational variance – due to unusual or unexpected occurrences

(c) The indexed principal in years 1 and 2 are as follows:

Indexed Principal1 = 1000 CPI1 / CPI0 = 1000 102 / 100 = 1020


Indexed Principal2 = 1000 CPI2 / CPI0 = 1000 106 / 100 = 1060

APMV Spring 2010 Solutions Page 16


8. Continued

Then the coupon payments in years 1 and 2 are:

Coupon Payment1 = Indexed Principal1 Real yield = 1020 0.03 = 30.6


Coupon Payment2 = Indexed Principal2 Real yield = 1060 0.03 = 31.8

(d) Indexed Principal3 = 1000 CPI3 / CPI0 = 1000 CPI3 / 100 = 10 CPI3

Coupon Payment3 = Indexed Principal3 Real yield = 10 CPI3 0.03 = 0.3


CPI3

Total Cash Flow at time 3 = Indexed Principal3 + Coupon Payment3


= 10 CPI3 + 0.3 CPI3
= 10.3 CPI3

Therefore, the cash flows for each of the three years of the bond are 30.6, 31.8,
and 10.3 CPI3.

Assuming a 7% realized nominal yield,

1000 = 30.6/(1.07) + 31.8/(1.07)2 + [10.3 CPI3]/(1.07)3

Solving the equation, we find that CPI3 = 112.2314.

APMV Spring 2010 Solutions Page 17


14. Learning Objectives:
(1d) Evaluate the particular issues influencing investment strategies for institutional
investors, including
Liquidity requirements,
Valuation concerns,
Cash flow variability,
Compliance risk,
Regulatory constraints,
Taxation impacts, and
Investment management mandates.

Sources:
V-C138-09 (Managing Your Advisor)

Liquidity risk Measurement – CIA Educational Note

Liquidity Modeling and Management, RSA, Volume 27, No. 2

Commentary on Question:
This is a recall, analysis and synthesis question asking candidates to evaluate investment
strategy for a product with liquidity optionality. The correct answers are expected to be
based on the content in study note V-C138-09 (Managing Your Advisor), CIA
Educational Note - Liquidity Risk Measurement, and Liquidity Modeling and
Management note from the Record of the Society of Actuaries, RSA Volume 27 No. 2.
Credit is given for answers that provide correct definitions of the risk, suggest an
appropriate investment policy for the product in question and outline a liquidity
management strategy consistent with the recommendations provided in the study notes.

Solution:
(a) Reference is CIA Educational Note – Liquidity Risk Management.

These are straightforward definitions taken directly from the study note. Poorer
candidates will often focus on market risk only, which is not the focus of the
question, or confuse the definitions of liquidity risk and liquidation risk.

Liquidity Risk is the inability to meet financial commitments as they fall due
through ongoing cash flow or asset sales at market value.

It is important to note that liquidity risk is distinct from market risk: liquidity risk
can exist without potential loss on sale of an asset (i.e. market risk – see next
item). While market risk is generally viewed as an asset issue, liquidity risk is an
asset/liability issue. The interaction of assets and liabilities is what matters (see
the RSA reference, page 3).

APMV Spring 2010 Solutions Page 28


14. Continued

Market Risk is the potential loss when the sale of an asset is required to fund the
cash demand. The loss could arise from deterioration in value of an asset due to
changes in interest rates, general market declines, decrease in credit quality of the
asset, or any other reason.

This risk is not to be confused with marketability risk, although marketability risk
is closely associated with the next item, liquidation risk.

Liquidation Risk is the potential loss when the sale of an asset is urgently required
which may result in the proceeds being below fair market value.

It is important to understand the distinction between liquidity risk and liquidation


risk. Liquidation risk can exist whether or not there is a need to meet financial
commitments (i.e. liquidity risk).

(b) References are Liquidity Modeling and Management (RSA), and Managing Your
Advisor.

The candidate is expected to critique the suggested strategy, i.e. discuss why the
various elements of the strategy are either appropriate or not appropriate in the
situation, rather than just identifying these elements.

Assets should be diversified along several dimensions to manage the liquidity


risks (RSA). While the student‟s suggested strategy has the potential to yield
gains, it comes at the increased cost of diversification, liquidity and
downgrade/spread risks, due to the concentration in lower investment grade assets
(i.e. along one dimension only).

Investing in zero-coupon bonds could create liquidity problems when


policyholders surrender their annuities and asset cash flows are not available to
match the liability. A laddered maturity structure (Managing Your Advisor, page
5) is preferable, minimizing the cash need surge for the company (RSA, page 8
paragraph 5).

BB spreads at the time of liquidation could be higher than 3%, since they are 3%
„on average‟. If spreads widen, the company would realize a loss in the event of
the asset sale.

It must be noted that the level of credit spreads and the change in these spreads
are the potential problem here, not necessarily the overall level of the yield curve
(although both are linked). The change in credit spreads introduces another
dimension to yield curve dynamics. A poorer candidate might concentrate on
yield movements without distinguishing interest rate levels and spreads.

APMV Spring 2010 Solutions Page 29


14. Continued

(c) Reference is Managing Your Advisor, which describes each of these investment
strategies.

A candidate is expected to understand and describe each of the strategies, being


able to compare them in order to ascertain which strategy is most appropriate for
the situation. It is expected that a candidate can demonstrate why each strategy
is more appropriate or less appropriate than the others. It is not enough to
simply choose one strategy and fit it into the given situation (e.g. by simply listing
the characteristics of the strategy that support the requirements or the situation).
Finally, a candidate is expected to unequivocally identify their preferred strategy.
Describing only one strategy without identifying this as the preferred strategy will
not be taken as a recommendation in and of itself. Recommendation of more than
one strategy is also not appropriate.

A laddered maturity is the recommended strategy.


Preferable to support portfolios with uncertain cash flows or policyholder
behavior, such as the non-forfeiture provision in the annuity.
Built with a bond maturing every few months, with asset portfolio cash flows
comprising coupon and principal payments.
Cash is readily available for benefit outflows and surrenders and to reinvest in
the current market rates for expected annuity cash flows.

In a bullet structure, asset cash flows are concentrated at one point.


More appropriate to support fixed and known benefit payments or lump sums,
not liabilities with policyholder optionality.
Could recommend cash flow matching with bullet bonds (more like an overall
laddered structure).
Generally more expensive than a laddered approach and in this case uncertain.

In Barbell structure, long and short duration assets are weighted to match an
immediate liability. This is a compromise between bullet and laddered
approaches.
Cash would be available from the short duration asset to support annuity
surrenders, although these are still uncertain.
Longer-duration asset will support policyholders who elect to keep the
annuity.

(d) Reference is CIA Educational Note – Liquidity Risk Management.

This is a straightforward list-type question taken directly from the study note,
allowing candidates to demonstrate that they are familiar with and understand
the syllabus material.

APMV Spring 2010 Solutions Page 30


14. Continued

Strategic Considerations:
Set corporate guidelines on short-term borrowings and/or asset mix; manage
overall balance sheet risk
Incorporate rating agencies‟ expectations
Address organizational level at which liquidity will be managed (total
company versus legal entity/segment)
Develop a contingency plan in the event of liquidity crisis

Operational Considerations:
ALM work closely with cash management function; define short- and
medium-term cash needs; possible impacts of different scenarios on liquidity
Assess how business trends and asset mix may impact liquidity in the future
(e.g. dynamic capital adequacy testing, other financial planning activities)
Thoughtful product design; how product features impact liquidity
Enforcement of various product features that impact liquidity

APMV Spring 2010 Solutions Page 31


22. Learning Objectives:
(3a) Critique and propose asset allocation strategies that can be used to construct an
asset portfolio.

Sources:
Ch. 9-10 and 26-28 of Litterman, Modern Investment Management.

V-C151-09 Sharpe, Expected Utility Asset Allocation

Maginn, Managing Investment Portfolios Ch. 8.

Tilman, Asset/Liability Management of Financial Institutions Ch. 6.

Commentary on Question:
This is a recall and comprehension question asking the candidates to demonstrate their
understanding of approaches used in developing a strategic asset allocation for a pension
plan, as described in Ch. 9-10 and 26-28 of Litterman, Modern Investment Management.

Solution:
(a) 1. The two points listed are correct. The following points could be added:
level of diversification
currency hedging
structure of active risk
duration matching.

Bond/equity split is the most important.

2. Bonds and public equities should be included. Consideration should be given


to domestic/foreign split. Long duration bonds should be added. Real estate
and commodities may be considered.
PE & HF may be inappropriate – high cost of due diligence and illiquidity.
PE may be inappropriate due to high risk.
On the other hand, HF may be appropriate – search for yield, diversification,
more mature HF market.
Since the plan in question is underfunded:
it may benefit from higher equity allocation as expected return on assets is
increased;
it may not benefit from global diversification;
there is greater benefit from duration matching.
Stocks & PE may be inappropriate – poor hedge for fixed-income liabilities.
HFs can be counted as floating asset that could be converted to fixed using
swaps, so consider swaps.

APMV Spring 2010 Solutions Page 47


22. Continued

3. This is questionable. EF results are very sensitive to assumptions. Historical


data is very sensitive to choice of historical period. Constraints may be
needed. There is potential for extreme portfolios.
There are issues with historical PE returns, lack of consistent historical HF
return data.
There are informational inefficiencies with HFs.
There is skewness in HF returns that is not captured in EF analysis.
It‟s important to take HF fees into account.
Historical returns are poor predictors of future returns. Expected returns
should be based on current conditions/typical relationships. Equilibrium
approach to setting expected returns is preferred.
Historical data may be robust enough to set risk characteristics (volatility &
correlations).
Benefits of the equilibrium approach: better neutral point, reliance on
observable info and identification of key trade-offs.
Could consider “reverse optimization” to set realistic assumptions.

4. Sharpe ratio good in the absence of liabilities, but not for a pension plan.
Better to optimize funded ratio or surplus.
Better measure: RACS (risk-adjusted change in surplus)
RACS = E[St+1 – St(1+Rf)]/ [St+1]
Where, St is surplus at time t and Rf is the risk-free rate.
Need qualitative evaluation.
Need to develop intuition for the structure.

(b) Should model liabilities, match duration.


Liability stream resembles a bond; hence liabilities could be modeled as a bond
portfolio.
Use current projected benefit payment information (cash flow) priced using
current term structure of interest rates.
Publicly traded bond index could be used as a proxy.
Index may be levered to match duration.
RL-Rf = (RB-Rf) + Where, RL is the return on liability index, Rf is the risk-free
rate, RB is the return on bond index.
is the duration matching parameter, is the noise term.
Additional sources of uncertainty: mortality rates, employee demographics and
salary growth.
Funding ratio low, so liabilities matter.
This is a mature plan; hence liquidity needs should be analyzed.
Consider dynamic analysis to investigate long-run impact of payouts.
Determine required returns.
Consider “hurdle rate” approach for HFs.
Consider expected utility asset allocation.
Consider a projection of outcomes over several years.

APMV Spring 2010 Solutions Page 48


23. Learning Objectives:
(2a) Describe and compare specialized financial instruments that can be used in the
construction of an asset portfolio supporting financial institutions and pension
plan liabilities.

(5f) Compare and select risk management techniques that can be used to deal with
financial and non-financial risks listed in (5b). (Currency risk, credit risk, spread
risk, liquidity risk, interest rate risk, equity risk, product risk, operational risk,
legal risk and political risk)

Sources:
Fabozzi, Handbook of Fixed Income Securities, 7th Edition, 2005 Ch.23, Agency
Mortgage-Backed Securities (pp. 513-527)

Babbel, D. and Fabozzi, F.J., Investment Management for Insurers Ch.20, Valuation and
Portfolio Risk Management with Mortgage-Backed Securities

Commentary on Question:
This question requires candidates to demonstrate a working knowledge of the features
and dynamics of Agency pass-through securities. Beyond a recitation of the features, the
question requires candidates to describe the importance of key features in the portfolio
management process, and to evaluate the appropriateness of several typical models that
are used to assess the appropriateness of these securities.

Solution:
(a)
(i) The WAC is important because it not only tells investors about the interest
rates of the underlying mortgages but also reveals the sensitivity of the
loan pool to prepayments.
When current mortgage rates available to borrowers are less than the
current WAC of the loan pool by 150 basis points or more, investors
would anticipate the pass-through to exhibit faster prepayment speeds.

(ii) WAM is another important measure because it gives investors an idea of


how many payments are remaining before the principal of the pool is
retired.
Specifically, the WAM represents an average maturity weighted by the
loan balances of the pool.
WALA is just the converse of WAM and represents the average age of the
underlying loans in the pool weighted by their balance.

APMV Spring 2010 Solutions Page 49


23. Continued

(b) 1. Turnover - prepayments that result from changing residences constitute a


base prepayment rate.
Due-on-sale clause: most conventional mortgages contain a due-on-sale
clause, which stipulates that the mortgage must be paid in full when the
house is sold.
Assumable: means the loan can be transferred to the new homeowner as
long as the new borrower meets minimum credit requirements.

2. Refinancing – represents the largest and most variable source of pre-


payments.
Borrowers with high credit quality who want to take advantage of lower-
cost mortgages in a falling-interest-rate environment are the key
refinancers.
Cash-out refinance: borrowers who refinance in order to borrow more
money than the existing loan balance on their property, provided that there
is sufficient equity in the house.
Credit curing: borrowers with previously tarnished credit histories are able
to refinance at more favorable rates because of improvements in their
credit ratings.

3. Defaults:
Not technically prepayments, but have the same financial effect as
prepayments in that the principal balance of the defaulted loan is returned
to the investor in the case of agency MBS.
Because of the explicit and implicit governmental guarantee provided to
the government-sponsored enterprises, the investor is protected from the
credit risk of individual borrowers that compose the pool.
Defaults of agency MBS represent only a small fraction of monthly
prepayments because of the high credit quality of the underlying
mortgages and therefore can be forecast as a component of prepayments.

(c)
(i) Static models:
Hedge against small changes from the current state of the world.
A term structure is an input to the model which matches assets and
liabilities under this structure.
The assets and liabilities will move in the same direction and by equal
amounts.
This is the fundamental principle behind portfolio immunization.

APMV Spring 2010 Solutions Page 50


23. Continued

(ii) Single-period, stochastic models:


Do not permit the specification of a stochastic process that describes
changes of the economic environment from its current status.
A stochastic ALM model describes the distribution of returns of both
assets and liabilities in the volatile environment and ensures that
movements of both sides of the balance sheet are highly correlated.
The implementation of this strategy requires the generation of
scenarios of holding period returns.
Does not account for the fact that the portfolio manager is likely to
rebalance the portfolio once some surplus is realized.
As the stochastic process evolves across time different portfolios may
be more appropriate for capturing the correlations of asset and
liability, the single-period model may recommend a conservative
strategy.

(iii) Multi-period, dynamic models:


This is a stochastic model of the previous section to a setting with
multiple time periods.
Assumes that a portfolio manager must make investment decisions
facing an uncertain future.
This model is substantially more flexible than the previous two
models.
This model explicitly allows for portfolio rebalancing at future time
periods, as more information about the uncertain scenarios becomes
available.
Transaction costs can also be incorporated.
By explicitly representing the scenario in the constraints we can
include scenarios not only of interest rates but also of prepayments,
spreads, etc.

APMV Spring 2010 Solutions Page 51


Questions 1 - 9 pertain to the Case Study
Each question should be answered independently.

6. (7 points) Wonka Life’s Chief Actuary, Wanda Fox, is concerned about the inflation-
linked aspect of Wonka Life’s Payout Annuity. Your money market trader says that
when inflation picks up, it is usually because the Fed has raised rates, and so suggests
investing in money market instruments as an inflation hedge. Wanda has asked you to
investigate using TIPS or money market investments to hedge this risk.

You are given the following information for a TIPS bond and NSA CPI-U Index levels
for the year 2010:

Issue Date 1/1/2010


Issue Price $100.00
Maturity 1 year
Real Coupon 6% paid semi-annually

Jan Feb Mar Apr May Jun Jul Aug Sep Oct Nov Dec
400.0 400.6 401.0 401.5 401.6 401.9 402.0 402.2 403.0 403.3 403.9 404.0

(a) (2 points) Calculate the settlement price on 6/10/2010 for this TIPS bond.

(b) (2 points) Discuss the approaches that can be used to quantify investment risk in
TIPS.

(c) (1 point) Evaluate using money market instruments to hedge the inflation-linked
aspect of the Payout Annuity line.

(d) (1 point) Evaluate using TIPS to hedge the inflation-linked aspect of the payout
line.

(e) (1 point) Recommend one of the two approaches and justify your recommendation.

Exam APM – Fall 2011 -6- GO ONTO NEXT PAGE


Advanced Portfolio Management
Morning Session
Questions 1 - 9 pertain to the Case Study
Each question should be answered independently.

9. (6 points) According to Wonka Life’s Asset Liability Management Report (December


31, 2009), the Traditional Life Product portfolio has a large duration mismatch that
concerns you. Since Wonka Life performs Asset-Liability Management on a segment by
segment basis, you are worried about a large floating rate position funding that liability
segment.

This floating rate security was created by splitting a fixed rate Treasury note into a floater
and inverse-floater.

Fixed rate Treasury:

Par value: 100 million


Coupon: 6% paid annually, fixed for life
Term: 3-years
Issue date: July 10, 2009
Options: None

Floater information:

Par value: 30 million


Reference: 1-year Treasury yield
Quoted spread: + 0.75 bps
Term: 3 years
Issue date: July 10, 2009
Coupons paid: Annually
Coupons reset: Annually
Options: None

(a) (1 point) Define the features of Floaters and Inverse Floaters.

(b) (2 points) Critique the appropriateness of using Floaters and/or Inverse Floaters
to support Wonka’s Traditional Life Product segment.

(c) (2 points) Calculate the price of the floating rate security assuming the annual
Treasury yield at issue on July 10, 2009 was 1.55%, as of:

(i) July 9, 2010

(ii) July 10, 2010

(d) (1 point) Calculate the July 10, 2010 coupon amount received from the inverse
floater.

Exam APM – Fall 2011 -9- GO ONTO NEXT PAGE


Advanced Portfolio Management
Morning Session
12. (14 points) You are an investment consultant assisting your client, University of
Actuaries, in reviewing the governance of its Endowment Fund. The Endowment Fund is
operated by a Trustee.

The goals of the Endowment Fund are to provide stable and sustainable funding to
support the University’s annual operating budget and to protect the donated capital on an
inflation-adjusted basis.

The spending rate has averaged 4% of market value of assets in recent years. The
University’s inflation rate has averaged 3% in recent years.

The recent financial crisis resulted in many problems for the University and its
Endowment Fund. Annual forecasted expenses are up, new donations are down and the
market value of assets ($450M) of the Endowment is now below the donated capital
($500M).

Although the University has no formal Statement of Investment Policy for the Fund, they
have established benchmark weights for performance monitoring purposes. You are
provided with the following information as of January 1, 2011:

Current Benchmark Expected


Allocation Allocation Return
Domestic Equity 25% 30% 8%
Foreign Equity 25% 30% 9%
Fixed Income 45% 30% 5%
Commodities 5% 10% 10%
Total 100% 100%

Standard Deviation of Return 8.5% 11%

(a) (3 points) Design a risk/return objective and constraints for the Endowment Fund
including a description of relevant factors.

(b) (2 points) Assess if the current benchmark allocation meets the risk/return
objective and recommend changes if necessary.

Exam APMV – Fall 2011 -2- GO ONTO NEXT PAGE


Advanced Portfolio Management
Afternoon Session
12. Continued

(c) (1 point) Explain the benefits and costs of rebalancing.

(d) (1 point) Assess different rebalancing strategies that could be considered for the
Endowment Fund.

(e) (2 points) Evaluate the role of commodities in the Endowment Fund portfolio.

(f) (2 points) Define the duties of the Trustee.

(g) (3 points) You estimate the Endowment Fund’s Board of Trustees risk aversion
( RA ) to be 4.

Determine whether the Current Allocation or the Benchmark Allocation would be


preferred if the decision is based on:

(i) Maximizing the expected utility for your client, or

(ii) Minimizing the probability of returns below 3%.

Exam APMV – Fall 2011 -3- GO ONTO NEXT PAGE


Advanced Portfolio Management
Afternoon Session
18. (6 points) You are the new advisor to the State of Columbia Lottery Commission, and
are required to formulate an investment strategy to fund the Commission’s required
payments over the next 20 years. The lottery agency is a state agency, which is exempt
from taxes, and has been directed to avoid any losses or shortfalls. The payments which
will be made at the end of each year, are as follows:

End of Year Payment


1 20 MM
2 19 MM
 
20 1 MM

(a) (2 points) Describe briefly how this liability schedule will impact considerations
of an investment policy.

(b) (2 points) Previously, the assets were invested in 5-year bonds, which are now
maturing, and will provide an immediate cash flow of 175 MM. You are
considering developing either a dedicated bond portfolio or active immunization.

For each of these two strategies:

(i) Describe the strategy.

(ii) Describe the advantages and disadvantages of using the strategy to fund
this particular liability stream.

(c) (2 points) In response to part (b) above, a dedicated bond portfolio was
developed. The cost of the portfolio was 180 MM. As an alternative to the
dedicated bond portfolio approach, which would require an additional 5 MM, the
prior advisor had suggested that a portion of the proceeds from the maturing
bonds be invested in emerging market debt.

(i) Describe the characteristics and risks of emerging market debt.

(ii) Describe the advantages and disadvantages of using this investment type
to fund this liability stream.

**END OF EXAMINATION**
Afternoon Session

Exam APM – Fall 2011 - 11 - STOP


Advanced Portfolio Management
Afternoon Session
6. Learning Objectives:
2. The candidate will understand the variety of financial instruments available to
managed portfolios.

Learning Outcomes:
(2a) Compare and select specialized financial instruments that can be used in the
construction of an asset portfolio supporting financial institutions and pension
plan liabilities.

Sources:
Fabozzi, Handbook of Fixed Income Securities, 7th Edition, 2005
 Ch. 15, “Inflation-Linked Bonds”

Commentary on Question:
Commentary listed underneath question component.

Solution:
(a) Calculate the settlement price on 6/10/2010 for this TIPS bond.

Commentary on Question:
Nearly nobody got this question right. The calculation is presented in 4 steps at
the top of page 355 of the HFIS, chapter 15, in a descriptive manner.

Due to the 3-month lag to incorporate CPI into TIPS indexation the July coupon
will be based on the April CPI level. Thus:
Inflation at 7/1/2010 = CPIapr / CPIjan - 1 = 0.375%
Notional at 7/1/2010 = Notional at 1/1/2010 x Inflation = $100 x 1.00375 =
$100.375
Coupon at 7/1/2010 = Notional at 7/1/2010 x coupon rate = $100.375 x (6%/2) =
$3.01125

Number of days in the first coupon period=181


Number of days from Jan 1, 2010 to June 10, 2010=160
Accrued Coupon= 3.01125*160/181=2.661878

To calculate the TIPS principal for any settlement other than the first of a month,
you need to interpolate between the CPI levels applicable to the beginning of the
month and the beginning of the next month.

Inflation applicable to 6/1/2010 = CPImar / CPIjan - 1 = 0.25%

APM Fall 2011 Solutions Page 15


6. Continued

Inflation applicable to 7/1/2010 = 0.375%


Notional at 6/1/2010 = $100.25
Notional at 7/1/2010 = $100 x 1.00375 = $100.38.
Notional at 6/10/2010 = Notional at 6/1/2010 + (9/30) x (Notional at 7/1/2010 –
Notional at 6/1/2010) = $100.25 + (9/30) x ($100.38 - $100.25) = $100.29
[For the fraction (9/30): 9 is the number of days of accrual (the 10th day of the
month – the 1st day of the month) and 30 is the number of days in the month]

Settlement prize= Accrued coupon + notional principle


Settlement prize= 100.29 + 2.661878=102.951878

(b) Discuss the approaches that can be used to quantify investment risk in TIPS.

Approaches that can be used to quantify investment risk in TIPS include:

The Real Duration

This is the percentage change in its market value associated with a 1.0% change
in its real yield. The formula is identical to that of a nominal bond. This does not
quantify the exposure of TIPS to changes in nominal yield. It is important as
nominal yield and real yield are correlated

The Effective Duration

This is calculated in practice by multiplying real duration by a factor of 75%.


This should only be used as loose metric for nominal interest rate exposure
because substantial risk (basis risk) remains. In theory TIP’s effective duration
measure risk as it relates to changes in nominal yield.

The Volatility

It is the simply the standard deviation of TIPS’ prices (or return).

The Relative Volatility

It represents the volatility as a fraction of the volatility of another instrument such


as a nominal bond having comparable maturity.

APM Fall 2011 Solutions Page 16


6. Continued

(c) Evaluate using money market instruments to hedge the inflation-linked aspect of
the Payout Annuity line.

Money Market securities are primarily short term assets maturing in one year or
less. The market is liquid and there is non-existant or low credit risk. This type
of asset is easy to invest in. The yield is expected to rise when inflation
expectations rise. There is no capital loss when rates rise. However, there is no
protection from unexpected inflation.

(d) Evaluate using TIPS to hedge the inflation-linked aspect of the payout line.

TIPs have built in inflation protection. The market is not very liquid.

Inflation-indexed bonds have notional amounts that increase with changes in the
consumer price index. Compared to money market they are bonds, which are
longer term investments. Therefore, a dedication strategy has to be considered to
pay out annuities. However, TIPS may overestimate expected inflation.

(e) Recommend one of the two approaches and justify your recommendation.

Annuity block contains some structured settlements with COLA escalators. The
COLA (cost of living adjustment) escalators index the benefit payment to
increases in the consumer price index.
The COLA escalators expose Wonka Life to high inflation rates. To hedge the
inflation risk, Wonka Life should invest assets in inflation-indexed bonds.

APM Fall 2011 Solutions Page 17


9. Learning Objectives:
2. The candidate will understand the variety of financial instruments available to
managed portfolios.

Learning Outcomes:
(2a) Compare and select specialized financial instruments that can be used in the
construction of an asset portfolio supporting financial institutions and pension
plan liabilities.

Sources:
HFIS Chapter 16

Commentary on Question:
Overall this question was poorly answered. This question takes information right out of
the textbook and should be well understood by the candidates.

Candidates appeared not to understand what duration was and we saw a number of
comments such as “Traditional Life Portfolio has a large duration but is not interest rate
sensitive.” This is a contradictory statement.

There was a typo in the question. The “Quoted spread” of the floater should have read
“75 bps” not “0.75 bps”. We accepted both answers since “0.75 bps” is not realistic and
the question was not intended to “trick” the candidate or test the candidate’s knowledge
of basis points.

Solution:
(a) Define the features of Floaters and Inverse Floaters.

Floaters
 Coupon interest varies over instrument's life
 Coupon formula = reference rate +/- quoted margin
 Typical reference rates
o LIBOR
o Treasury bill yields
o Prime rates
o Domestic CD rates
 Coupon moves in direction of the reference rate
 DURATION is typically small

Inverse Floaters
 General formula: coupon = K - L x quoted margin
 Typically very large DURATION
 Coupon varies inversely with reference rate

APM Fall 2011 Solutions Page 24


9. Continued

(b) Critique the appropriateness of using Floaters and/or Inverse Floaters to support
Wonka’s Traditional Life Product segment.

Commentary on Question:
A lot of Candidates did not look at the Traditional Life segment from an interest
rate risk perspective. The question clearly talks about Duration Mismatch and not
cash flow matching, yet a number of candidates answered based on a cash flow
matching type of management.

Also, a candidate needs to clearly explain what they mean by “interest rate
sensitive.” Many candidates would say the floater is very interest rate sensitive.
This is true if you are describing the cash flows of the floater, but if you are
describing the market value of the floater, this statement is just wrong. Interest
rate sensitivity is generally measured by duration and usually refers to the market
value of the asset.

Portfolio Observations
 Asset duration is shorter than liability duration of portfolio
 Asset duration is approximately 7.8 years
 Need to purchase assets longer than 7.8 years to lengthen asset duration

Floater Appropriateness
 Very short durations
 Inappropriate if we want to lengthen duration

Inverse-floater Appropriateness
 Very long durations
 May be appropriate for lengthening duration

(c) Calculate the price of the floating rate security assuming the annual Treasury
yield at issue on July 10, 2009 was 1.55%, as of:

(i) July 9, 2010

Commentary on Question:
There were two ways to solve this using PV of future cash flows. The first
method is recognizing that the price of a floater is just the PV of the
coupon and par value from the next reset date. The second method
involves understanding which discount rate to use to discount all future
cash flows.

This question was meant to test the candidate’s understanding of how the
price of a floater changes during reset periods before the coupon has been
paid and at the reset date after the coupon has been paid.
APM Fall 2011 Solutions Page 25
9. Continued

First Solution

Coupon = $30M x .(1.55% + 0.75%) = $0.69M (has not been paid yet)

Time to reset is 1 day. (negligible time so price is just the cash flow plus
par value at the next reset date)

Price = Par + Coupon


Price = 30M + 0.69M
Price = 30.69

Second Solution
CF t 0.69 30  0.69
Price =   0.69    30.69
1  i t
1.023 1.0232
i = current floater rate = 1.55% + 0.75% = 2.30%
CFt = 0.69 (for non-maturity date)
CFt = 30.69 (at maturity)

(ii) July 10, 2010

Both methods described in (i) can be used here. Since the coupon has
already been paid the price is simply 0.69 less than part (i).

Price = 30

(d) Calculate the July 10, 2010 coupon amount received from the inverse floater.

Commentary on Question:
This was an extremely easy question. We were surprised not more people were
able to calculate it. The most common mistake was people tried to add the yields
of the Floater and Inverse Floater instead of the actual dollar amounts.

Fixed Bond Coupon ($) = Floater Coupon ($) + Inverse Floater Coupon ($)

Fixed Bond Coupon = 100 x 6% = 6


Floater Coupon = 30 x 2.30% = 0.69

Inverse Floater Coupon = 6 – 0.69 = 5.31

APM Fall 2011 Solutions Page 26


12. Continued

The Duty to Act in Accordance with the Trust Agreement


Obligation to manage the trust in accordance with the trust agreement. Trustee
should not engage in a transaction that breaches another duty to the detriment of
the participants and beneficiaries.

(g) Determine whether the Current Allocation or the Benchmark Allocation would be
preferred if the decision is based on:

(i) maximizing the expected utility for your client, or

UX = E(RX) – 0.005*RA*σ(RX)^2 = 7 – 0.005 * 4 * 8.5^2 = 5.55


UY = E(RY) – 0.005*RA*σ(RY)^2 = 7.6 – 0.005 * 4 * 11^2 = 5.18
Allocation X has higher risk-adjusted expected return, therefore should
choose X (current allocation).

(ii) minimizing the probability of returns below 3%.

Use Roy’s safety-first criterion with a return threshold (RL) of 3%


Roy’s SFRX = (E(RX) – RL)/ σ(RX) = (7 – 3 )/8.5 = .47
Roy’s SFRY = (E(RY) – RL)/ σ(RY) = (7.6 – 3 )/11 = .42
should choose allocation with the largest SFR, should choose X (current
allocation)

APM Fall 2011 Solutions Page 37


18. Learning Objectives:
2. The candidate will understand the variety of financial instruments available to
managed portfolios.

5. The candidate will understand the specific considerations relative to managing a


fixed income portfolio within an asset allocation framework.

Learning Outcomes:
(2a) Compare and select specialized financial instruments that can be used in the
construction of an asset portfolio supporting financial institutions and pension
plan liabilities.

(5a) Explain how an investment policy affects the selection of an investment strategy
or the selection of an optimal portfolio.

(5d) Recommend an investment strategy for a given situation:


 Portfolio policy and objectives
 Asset selection criteria
 Capital market expectations
 Risk management strategy
 Portfolio rebalancing strategy

Sources:
Formation of Investment Strategy for Insurance Companies and Pension Plans

HFIS, Chapter 20, Emerging Market Debt

HFIS, Chapter 48, Dedicated Bond Portfolios

CFA Textbook Chapter 1

Commentary on Question:
Most candidates did well describing the considerations of an investment policy in (a) and
describing the strategies given in part (b). Many struggled to describe characteristics of
emerging market debt in part (c).

Solution:
(a) Describe briefly how this liability schedule will impact considerations of an
investment policy.

Considerations for the investment policy:


 Liability characteristics: cash flows are higher in the early years, meaning
short duration investments may be appropriate.
 Liquidity: the state will need sufficient liquidity to make payments when they
are due.

APM Fall 2011 Solutions Page 52


18. Continued

 Risk tolerance: the investment should be low risk, as the state must avoid
losses and shortfalls.
 Return objective: the investment must provide sufficient returns to cover all
future payments.
 Legal and regulatory factors: the agency is tax exempt so taxes are not a factor
in the strategy.

(b) For each of these two strategies:

(i) Describe the strategy.

Dedicated Bond Portfolio:


 Portfolio assets cover liability cash flows for each time period

Active Immunization:
 Duration of asset portfolio is matched to liability portfolio

(ii) Describe the advantages and disadvantages of using the strategy to fund
this particular liability stream.

Dedicated Bond Portfolio:


Advantage: Locks in price with very little risk
Disadvantage: Larger time and effort to execute

Active Immunization:
Advantage: Allows active management of portfolio
Disadvantage: Requires ongoing resources to maintain

(c) In response to part (b) above, a dedicated bond portfolio was developed. The cost
of the portfolio was 180 MM. As an alternative to the dedicated bond portfolio
approach, which would require an additional 5 MM, the prior advisor had
suggested that a portion of the proceeds from the maturing bonds be invested in
emerging market debt,

(i) Describe the characteristics and risks of emerging market debt.

Characteristics of emerging market debt:


 Less liquid
 Varied practice by country
 Most debt issued by Asian or Latin American countries
 Ratings have improved, wide range of credit quality available

APM Fall 2011 Solutions Page 53


18. Continued

Risks:
 More volatile asset class
 Other risks – currency, political, default, etc.

(ii) Describe the advantages and disadvantages of using this investment type
to fund this liability stream.

Advantages
 Higher yield to cover the shortfall
 Could select bonds from different countries that may not be positively
correlated, which in turn would enhance diversification

Disadvantages
 Cash flows may be volatile, and may not coincide with the liability
stream

APM Fall 2011 Solutions Page 54


**BEGINNING OF EXAMINATION**
Morning Session

Questions 1- 6 pertain to the Case Study


Each question should be answered independently.

1. (6 points) Wonka Life is offering a universal life product that has an account value with
a credited rate based on Wonka’s portfolio rate, subject to a guaranteed minimum of 4%
per year. The product has a decreasing surrender charge which is deducted from the
account value if the policy is surrendered.

(a) (2 points) Evaluate the current investment strategy for the universal life product
with respect to interest rate risk.

(b) (2 points) Describe the advantages and disadvantages of using callable corporate
bonds to back this block of business.

(c) (2 points) Recommend an investment strategy to better manage Wonka’s


exposure to interest rate risk for the universal life product and justify your
recommendation.

Exam APM – Spring 2011 -1- GO ONTO NEXT PAGE


Advanced Portfolio Management
Morning Session
Questions 1- 6 pertain to the Case Study
Each question should be answered independently.

2. (7 points)

(a) (1 point) Describe considerations when calculating the effective duration for a
portfolio of non-callable corporate bonds.

(b) (1.5 points) Describe how the effective duration of Wonka’s Universal Life
product will be affected by a decrease in interest rates.

(c) (0.5 points) Describe the risks of investing in agency mortgage pass-throughs to
back Wonka Life’s UL products.

(d) (2 points) Calculate the effect of issuing $2 billion in new insurance liabilities
on Wonka Life’s surplus duration. Assume any new business written and assets
invested will have the same effective durations as the existing business and
investments, respectively, and that new sales have no surplus strain.

(e) (1 point) Recommend a method for managing the surplus duration back to
within guidelines.

(f) (1 point) Explain whether it is possible to manage the asset duration to achieve a
zero surplus duration.

Exam APM – Spring 2011 -2- GO ONTO NEXT PAGE


Advanced Portfolio Management
Morning Session
Questions 1- 6 pertain to the Case Study
Each question should be answered independently.

6. (5 points) Recommend risk reduction strategies that would help Wonka Life in
managing the following risks of their Employees’ Pension Plan, and justify your
recommendations.

(i) Longevity Risk

(ii) Inflation Risk

(iii) Liquidity Risk

(iv) Currency Risk

(v) Pension Funding Risk

7. (4 points) Your company has three investment portfolios. Your company’s guideline on
single company exposure mandates that credit risk exposure to a single issuer within a
portfolio be limited. The managers of the three portfolios are reviewing alternative ways
of managing issuer-specific risk.

Portfolio/Manager A B C
Portfolio Size Large Small Small
Credit Research Ability Weak Strong Strong
Transaction Cost Efficiency Efficient Not Efficient Not Efficient
Complies with Guideline on
Yes Yes No
Single-Company Exposure

(a) (1 point) Explain how the manager of Portfolio C can use credit default swaps
(CDS) to manage single issuer risk, while providing a similar cash flow pattern
and achieving the original target total return.

(b) (3 points) Recommend approaches to manage issuer-specific risk other than


using CDS to the managers of Portfolio A and B. Justify your recommendations.

Exam APM – Spring 2011 -5- GO ONTO NEXT PAGE


Advanced Portfolio Management
Morning Session
9. (5 points) Culebra Life has just acquired a large block of liabilities. This new
acquisition has shifted the liability DV01 significantly and it is now 850,000.

Culebra’s Asset Portfolio


Duration Spread Duration Book Value Market Value Current
Asset Class (years) (years) (’000s) (’000s) yield
Cash 0 0 250,000 240,000 1.00%
Treasury Bonds 11.0 0 250,000 275,000 4.00%
Corporate Bonds 6.2 5.9 400,000 500,000 5.50%
Mortgages 2.2 1.3 500,000 475,000 5.00%
Total 1,400,000 1,490,000

(a) (2 points)

(i) Define spread duration.

(ii) Describe the major types of spread duration.

(b) (3 points) The CFO has expressed concern that the current portfolio yield is too
low, but Culebra’s investment policy restricts spread duration to be no more than
3.0 years.

(i) Calculate the maximum allowable allocation to Corporate Bonds under


the investment policy while keeping the allocations to Mortgages at the
current level.

(ii) Recommend an asset allocation that rebalances the portfolio so that the
asset dollar duration matches the liability while being sensitive to the
CFO’s concerns. Justify your recommendation.

Exam APM – Spring 2011 -7- GO ONTO NEXT PAGE


Advanced Portfolio Management
Morning Session
13. (5 points) You are given the following information about the components of an equity
market segment:

Share Price Share Price Market Value Market Value Free Float
Company
12/31/2010 12/31/2011 12/31/2010 12/31/2011 Factor
YourWay Airways 14 8 4,018 2,296 1.00
Snack International 15 25 19,035 31,726 1.00
RugWorth Retail 10 11 43,950 48,345 0.60
Import International 14 17 150,052 182,206 0.85
Total 217,055 264,573

(a) (2 points) Describe the characteristics of four common index weighting methods
for this equity market segment.

(b) (2 points) Calculate the return of this market segment for each of the four index
weighting methods identified in part (a).

(c) (1 point) Recommend a benchmark that is consistent with a passive management


strategy of investing in this market segment and justify your recommendation.

Exam APM – Spring 2011 -2- GO ONTO NEXT PAGE


Advanced Portfolio Management
Afternoon Session
16. (10 points) The fund manager of the Pennywise Pension Plan (a defined benefit pension
plan) is choosing between Portfolio A and Portfolio B to back the liabilities of the Plan.

(a) (0.5 points) Explain the differences between the Asset/Liability Management
(ALM) approach and the asset-only approach to the strategic asset allocation
process.

(b) (0.5 points) Describe situations where an investor would favor the ALM
approach.

(c) (1 point) Compare the appropriateness of the traditional Sharpe Ratio measure
and the Risk Adjusted Change in Surplus (RACS) when managing the assets of
this pension plan.

(d) (4 points) You are given the following information:

Pension
Portfolio A Portfolio B
Liabilities
Expected Annual
6% 5% 4%
Rate of Growth
Standard
Deviation of the 12% 11% 10%
Rate of Growth
Correlation
Coefficient with 0.2 0.9 1
Liabilities
Value at Year-End
110 110 100
2009

The risk-free rate is 3% per year.

(i) Calculate the RACS for each of Portfolio A and Portfolio B.

(ii) Recommend which of Portfolios A or B should be selected, based only


on the RACS.

Exam APM – Spring 2011 -7- GO ONTO NEXT PAGE


Advanced Portfolio Management
Afternoon Session
16. Continued

(e) (4 points) In addition to the information provided in (d), assume that the
Pennywise Pension Plan has a payout rate of 8% per year.

(i) Calculate the funded ratio for the Pension Plan at year-end 2009.

(ii) Calculate the minimum excess return over liabilities that the fund must
achieve in order to maintain the same funded ratio throughout 2010.

(iii) Recommend a strategic asset allocation strategy, given that the Pension
Plan is overfunded, and justify your recommendation.

Exam APM – Spring 2011 -8- GO ONTO NEXT PAGE


Advanced Portfolio Management
Afternoon Session
17. (8 points) ABC Company has decided to introduce a defined contribution (DC) pension
plan. A member of ABC Company’s pension committee has suggested making the
following investment options available to plan members for their self-directed accounts:

(i) Money Market Fund

(ii) Large Cap Equity Fund

(iii) Venture Capital Fund

(iv) Commodity Index Fund

(a) (1 point) Describe the purpose of establishing a Statement of Investment Policy


in the context of a DC pension plan.

(b) (1 point) Identify the responsibilities of ABC Company in developing the list of
investment options for a DC pension plan.

(c) (1 point) Describe the behavioral biases facing DC plan members.

(d) (2 points) Assess the appropriateness of each of the investment options suggested
by the committee member.

(e) (2 points) The Company wishes to offer only four investment options.

Recommend changes to the above list of investment options and justify your
recommendation.

(f) (1 point) Recommend a default investment option for members who refrain from
selecting any options and justify your recommendation.

Exam APM – Spring 2011 -9- GO ONTO NEXT PAGE


Advanced Portfolio Management
Afternoon Session
APM Complete Illustrative Solutions
Spring 2011

1. Learning Objectives:
2. The candidate will understand the variety of financial instruments available to
managed portfolios.

Learning Outcomes:
(2a) Compare and select specialized financial instruments that can be used in the
construction of an asset portfolio supporting financial institutions and pension
plan liabilities.

Sources:
Fabozzi, Handbook of Fixed Income Securities, 7th edition, 2005
• Chapter 10, U.S. Treasurey and Agency Securities (pgs. 229-231, 241-245)
• Chapter 13, Corporate Bonds (pgs 305-327, 331-335)
• Chapter 14, Medium-Term Notes (pgs 339-340, 344-350)

Commentary on Question:
This is a recall and synthesis question asking candidates to (a) evaluate the risks inherent
in a given investment strategy for a given insurance liability structure, (b) apply their
knowledge of corporate callable bonds and evaluate their appropriateness in backing an
interest-sensitive liability structure, and (c) formulate an appropriate investment strategy
for a specific product line and explain why the strategy is appropriate for that product
line.

Solution:
(a) Evaluate the current investment strategy for the universal life product with respect
to interest rate risk.

Wonka Life has a severe duration mismatch between assets and liabilities of 5.3
years, and a dollar duration mismatch of over $2 million which exceeds the
guideline of $0.4 million. This exposes the company to significant interest rate
risk.
The current investment strategy subjects Wonka Life to risks in rising interest rate
environments, because securities may have to be liquidated at a capital loss to
cover cash surrender value.
Wonka Life is also subject to reinvestment risk due to the current low interest rate
environment. The strategy does not provide a sufficient hedge on interest rate risk
due to the minimum product guarantee.

APM Spring 2011 Solutions Page 1


1. Continued

(b) Describe the advantages and disadvantages of using callable corporate bonds to
back this block of business.

Advantage:
Callable corporate bonds generally have a higher yield than comparable
government bonds so as to compensate for the call risk. These bonds also have a
shorter duration which better aligns to the company’s liability duration.

Disadvantage:
Bonds can be called prior to the maturity or termination of the insurance contract.
This may introduce uncertainty of asset cash flows and provide insufficient cash
flows to fund those required by surrenders and terminations.
Moreover, bonds are more likely to be called as interest rates decline, which
exposes Wonka Life to reinvestment risk. It would be difficult to find a bond
yielding more than the minimum crediting interest guarantee. Unlike Treasury
securities, corporate callable bonds may have higher degrees of default risks.

(c) Recommend an investment strategy to better manage Wonka’s exposure to


interest rate risk for the universal life product and justify your recommendation.

The strategy must address the growing duration mismatch between assets and
liabilities. Wonka Life can consider rebalancing the portfolio by investing in
shorter maturity assets, or entering into an interest rate swap or other types of
financial instruments such as interest rate floors and floaters.

APM Spring 2011 Solutions Page 2


2. Continued

(c) Describe the risks of investing in agency mortgage pass-throughs to back Wonka
Life’s UL products.

Agency mortgage pass-throughs have prepayment risk, where prepayments


increase with lower interest rates. The effective duration therefore decreases with
lower interest rates.
The asset duration therefore moves in opposite directions with the liability
duration when interest rates decrease, presenting a significant asset liability
mismatch.

(d) Calculate the effect of issuing $2 billion in new insurance liabilities on Wonka
Life’s surplus duration.

Ds = (Da – Dl) * A/S + Dl


Da = 6. DI = 5.3
With new debt:
PV Assets = 6,031,012 + 2,000,000 = 8,031,012
PV Liabilities = 5,220,000 + 2,000,000 = 7,220,000
Surplus is unchanged at 807,912
New Leverage A/S = 8,031,012/807,912 = 9.94
"New Surplus Duration = 12.26
Surplus duration has increased. Existing duration was 10.6

(e) Recommend a method for managing the surplus duration back to within
guidelines.

The current surplus duration is larger than the guidelines. In order to reduce gap,
a number of actions can be taken:
• Rebalance into shorter duration assets (e.g. shorter maturity corporate bonds,
floating rate securities)
• Use derivatives to reduce the asset duration
• The leverage could be decreased by divesting blocks of businesses
• The liability duration can be modified by changing product characteristics of
inforce products, or more likely, for new business

(f) Explain whether it is possible to manage the asset duration to achieve a zero
surplus duration.

This is theoretically possible, though practically difficult. From the formula we


see that:
Ds = (Da – Dl) * A/S + Dl
0 = (Da-DI)*A/S + DI
Da = DI(A/S-1)*S/A
So we need to set the asset duration equal to the above amount. This is
effectively matching the dollar duration of assets with dollar duration of liabilities

APM Spring 2011 Solutions Page 4


6. Learning Objectives:
1. Candidate will understand and be able to follow the investment management
process for insurance companies, pension funds and other financial
intermediaries.

2. The candidate will understand the variety of financial instruments available to


managed portfolios.

4. The candidate will understand the specific considerations relative to managing an


equity and/or alternative asset portfolio within an asset allocation framework.

Learning Outcomes:
(1a) Explain how an investment policy and an investment strategy can help manage
risk and create value.

(1c) Determine how a client’s objectives, needs and constraints affect the selection of
an investment strategy or the construction of a portfolio. Considerations include:
• Funding objective
• Risk-return trade-off
• Regulatory and rating agency requirements
• Risk appetite
• Liquidity constraints
• Capital, tax and accounting considerations

(1d) Identify and describe the impact on investment policy of financial and non-
financial risks including but not limited to: Currency risk, credit risk, spread risk,
liquidity risk, interest rate risk, equity risk, product risk, operational risk, legal
risk and political risk.

(2a) Compare and select specialized financial instruments that can be used in the
construction of an asset portfolio supporting financial institutions and pension
plan liabilities.

(4a) Explain how an investment policy affects the selection of an investment strategy
or the selection of an optimal portfolio.

Sources:
Liability-Relative Strategic Asset Allocation Policies. Pages 57 to 59

“Living with Mortality: Longevity Bonds and Other Mortality-Linked Securities”, by


Blake, Cairns and Dowd, Institute of Actuaries, 2006 (Sections 3-5)

Balancing the opportunities in real return investments by Robert Bertram page 45-45-47

Liquidity Risk Measurement - CIA Educational Note

APM Spring 2011 Solutions Page 11


6. Continued

Derivatives: Practices and Principles. Page 37

Liability-Relative Strategic Asset Allocation Policies. Pages 44 and 56

Commentary on Question:
The candidates were expected to provide recommendations on strategies that could be
used to mitigate various risks facing pension plans.

Solution:
Recommend risk reduction strategies that would help Wonka Life in managing the
following risks of their Employees’ Pension Plan, and justify your recommendations.

Longevity Risk: They can diversify their longevity risks. Balance their portfolio by
seeking to exploit possible natural hedges involved running a mixed business of term
assurance and annuity business. They can enter into a variety of forms of reinsurance
with a reinsurance company. They can manage the risk using mortality-linked securities.
These securities might be traded contracts (longevity bonds, annuity future, options, etc.)
or over-the-counter contracts (mortality swaps or forwards).

Inflation Risk: Invest in TIPS or real return bonds. Invest in commodities. Commodities
have the lowest correlation with stock market activity, so they are a good diversifier that
provides a hedge against unexpected inflation. Invest in real estate portfolio. Real estate
offers long term average returns of the CPI plus 5% and it also offers good long term
protection against inflation. Invest in infrastructure portfolio. Infrastructure assets offer
stable returns and pass through unanticipated inflation. Invest in timberland. Timberland
is a good hedge against unexpected inflation. Returns are largely depending on organic
growth. Increase exposure to equities. Inflation factors are best modeled as equity-like
exposures. To properly hedge those exposures, plan needs some portion of the fund in
equities.

Liquidity Risk: Invest in cash or other short-term assets (do not involve significant credit
risk). Increase exposure to fixed income (e.g. bonds). Public bonds are more liquid that
non-investment grade. Invest in marketable instruments where there is an active
secondary market. Increase contribution.

Currency Risk: Enter into currency swaps and futures/forwards and other derivatives can
be used to eliminate currency risk. Reduce allocation to foreign equities and fixed
income.

APM Spring 2011 Solutions Page 12


6. Continued

Pension Funding Risk: Pension funding risk is controlling the net of the assets and the
liabilities (the deficit and surplus). The pension Plan is actually underfunded (Assets less
than Liabilities). Investment policy, contribution policy and benefit policy are all very
important and should be reviewed in a holistic manner in order to control pension funding
risks. Plan sponsor should focus on liability-relative investment policies and approaches
(liability matching portfolios). Plan sponsor should consider controlling volatility of
surplus and contribution and expense through liability-relative investment policies such
as interest rate hedging using swaps or other derivative instruments. Plan sponsor needs
to review the allocation to risky assets to find an appropriate balance between the growth
of the plan surplus and the risk to the funded status. A consideration might be given to
reducing the allocation to risky assets.

APM Spring 2011 Solutions Page 13


7. Learning Objectives:
7. The candidate will understand the purposes and methods of portfolio performance
measurement.

Learning Outcomes:
(7a) Describe and assess performance measurement methodologies for investment
portfolios.

(7b) Describe and assess techniques that can be used to select or build a benchmark for
a given portfolio or portfolio management style.

(7c) Recommend a benchmark for a given portfolio or portfolio management style.

(7d) Recommend a performance measurement methodology.

Sources:
Fabozzi HFIS Chapter 44 – Quantitative Management of Benchmarked Portfolios

Commentary on Question:
This question focuses on managing issuer-specific risk in fixed income portfolios.

Solution:
(a) Explain how the manager of Portfolio C can use credit default swaps (CDS) to
manage single issuer risk, while providing a similar cash flow pattern and
achieving the original target total return.

The manager can use Credit Default Swaps to remove the existing exposures in
the portfolio and to create new exposure that is suitable for benchmark matching
without violating the single-name policy.

They can enter into CDS as the protection buyer, use CDS to hedge the part of
over-exposure to specific single issuer.

For the exposures that are not over the exposure limit, they can consider selling
some CDS protection to generate some income to cover the CDS spread they need
to pay for the CDS protection.

(b) Recommend approaches to manage issuer-specific risk other than using CDS to
the managers of Portfolio A and B.

For Portfolio A:
I suggest the manager to reduce the issuer –specific risk by using diversification
in its credit portfolio.

Relatively larger portfolio size and efficient transaction cost management will
mitigate the higher transaction cost due to the frequent rebalancing requirement.

APM Spring 2011 Solutions Page 14


7. Continued

Portfolio A does not have a dedicated credit research resource. Managers will be
forced to extend to issuers that are not highly rated by their credit analysts and
therefore dilute the value of the credit research.

For Portfolio B:
I recommend using swaps as a total-return investment.

It does not require cash up front, so it is suitable for smaller funds with transaction
cost constrains.

APM Spring 2011 Solutions Page 15


9. Learning Objectives:
5. The candidate will understand the specific considerations relative to managing
affixed income portfolio within an asset allocation framework.

Learning Outcomes:
(5b) Assess a portfolio position against portfolio management objectives using
qualitative and quantitative techniques.

(5f) Demonstrate how to apply funding and portfolio management strategies to control
interest rate and credit risk, including key rate risks.

Sources:
Managing Investment Portfolios, Chapter 6, Section 4.1, 5.3

Commentary on Question:
This is a recall and application question that requires candidates to (a) provide
information about the definition and types of spread duration and (b) determine the asset
allocation that will satisfy stipulated criteria.

Solution:
(a)
(i) Define spread duration.

Measure of how the market value of a risky portfolio changes with a


parallel 1% shift in spread above the benchmark portfolio.

(ii) Describe the major types of spread duration.

Nominal Spread
Spread of a bond or portfolio above the yield of a certain maturity
Treasury

Static Spread or Zero-Volatility Spread


Defined as the constant spread above the Treasury spot curve that equates
the calculated price of the security to the market price

OAS – Option Adjusted Spread


Current spread over the benchmark yield minus that component of the
spread that is attributable to any embedded optionality in the instrument

(b) The CFO has expressed concern that the current portfolio yield is too low, but
Culebra’s investment policy restricts spread duration to be no more than 3.0 years.

(i) Calculate the maximum allowable allocation to Corporate Bonds under the
investment policy while keeping the allocations to Mortgages at the
current level.

APM Spring 2011 Solutions Page 18


9. Continued

Spread Duration for a portfolio is the market weighted average of the


spread durations of the component securities.

In order to keep the allocations to Mortgages the same, we must use cash
and/or sell Treasury bonds to purchase more Corporate Bonds

Let b be the market value of total Corporate bonds (in Millions) in the
resultant portfolio. Then,

{(1015 – b)*0 + b *5.9 + 475 * 1.3} / 1490 = 3


b = 653

The maximum allocation to Corporate bonds is 653 million.

(ii) Recommend an asset allocation that rebalances the portfolio so that the
asset dollar duration matches the liability while being sensitive to the
CFO’s concerns.

Dollar Duration of portfolio is:


(0*240 Million + 11 * 275 Million + 6.2 * 500 Million + 2.2 * 475
Million)/100 = 71.7 million

To increase the Dollar Duration to 850,000 * 100 = 85 million one would


increase the allocation to Treasury Bonds while reducing cash.

This will increase dollar duration while limiting the spread duration to the
desired target.

This addresses the CFO’s concerns and manages spread duration


restriction within Culebra’s investment policy.

APM Spring 2011 Solutions Page 19


13. Learning Objectives:
4. The candidate will understand the specific considerations relative to managing an
equity and/or alternative asset portfolio within an asset allocation framework.

Learning Outcomes:
(4b) Assess a portfolio position against portfolio management objectives using
qualitative and quantitative techniques.

Sources:
Maginn & Tuttle, Managing Investment Portfolios 3rd Ed. Equity Portfolio Management,
Pgs. 353 and 357

Commentary on Question:
This is a recall and calculation question testing candidates’ understanding of equity index
construction methodology.

Solution:
(a) Describe the characteristics of four common index weighting methods for this
equity market segment.

Price-Weighted Index
Each stock is weighted according to its absolute value.
Sum of the shares prices divided by the adjusted number by shares in the index.
Represents the performance of a portfolio that simply bought and held one share
of each index component.

Value-Weighted Index (or Market Capitalization)


Each stock in the index is weighted according to its market cap.
Share price multiplied by the number of shares outstanding.
Represents the performance of a portfolio that owns all the outstanding shares in
each index component.
Self corrects for stock splits, reverse stock splits, and dividends because such
actions are directly reflected in the number of shares outstanding and price per
share for the company affected.
Biased to the shares of companies with the largest market capitalization.

Float-Weighted Index
Each stock in the index is weighted according to the number of shares outstanding
that are actually available to investors.
Represents the performance of a portfolio that bought and held all the shares of
each index components that are available for trading.
The weight of a stock in a float-weighted index equals its market-cap weight
multiplied by a free-float adjustment factor.

APM Spring 2011 Solutions Page 26


13. Continued

Equal-Weighted Index
Each stock is weighted equally.
Represents the performance of a portfolio in which the same amount of money is
invested in the shares of each index component.
Must be rebalanced periodically to reestablish the equal weighting.

(b) Calculate the return of this market segment for each of the four index weighting
methods identified in part (a).

Price-Weighted Index
Average of share Price December 31, 2010 / 453 / 4 = 13.25
Average of Share Price December 31, 2011 / 461 / 4 = 15.25

Value-Weighted Index (or Market Capitalization)


Market Value December 31, 2010 217,055
Market Value December 31, 2011 264,573
Value-Weighted Index return = 264,573 / 217,055 – 1
2189%

Float-Weighted Index
Sum(Market Value December 31, 2010 x Free Float Factor) = 176,967
Sum(Market Value December 31, 2011 x Free Float Factor) = 217,904
Float-Weighted index return =217,904 / 176,967
23.13%

Equal-Weighted Index
Step 1: Calculate Price Change of each Share

YourWay Airways -42.86%


Snack International 66.67%
RugWorth Retail 10.00%
Import International 21.43%

Step 2: Equally weigh each price change or take the average

Equal-Weighted Index Return = Average (-42.86%, 66.67%, 10.00%, 21.43%)


13.81%

(c) Recommend a benchmark that is consistent with a passive management strategy


of investing in this market segment and justify your recommendation.

A float- weighted index of the six shares is the recommended benchmark index.
It reflects the supply of shares actually available to the public.

APM Spring 2011 Solutions Page 27


16. Learning Objectives:
3. The candidate will understand the importance of the techniques and theory behind
portfolio asset allocation.

Learning Outcomes:
(3c) Evaluate the significance of liabilities in the determination of the asset allocation.

Sources:
Litterman, Modern Investment management: An Equilibrium Approach, 2003
• Chapter 10, Strategic Asset Allocation in the Presence of Uncertain Liabilities

Maginn & Tuttle, Managing Investment Portfolios, 3rd Ed. 2008


• Chapter 5, Asset Allocation

Commentary on Question:
This question is asking the candidates to demonstrate their knowledge of methods that
can be used in the determination of the asset allocation for a pension plan.

Solution:
(a) Explain the differences between the Asset/Liability Management (ALM)
approach and the asset-only approach to the strategic asset allocation process.

The ALM approach involves explicitly modeling liabilities and adopting the
optimized asset allocation in relationship to funding liabilities.
The Asset-only approach does not explicitly involve modeling liabilities.
The ALM approach typically results in a higher allocation to the fixed-income
instruments than an Asset-only approach.
Fixed-income instruments have pre-specified interest/principal payments that
typically represent legal obligations of the issuer. Because of the nature of their
cashflows, fixed-income instruments are well suited to offsetting future liability
obligations.

(b) Describe situations where an investor would favor the ALM approach.

The investor has below-average risk tolerance.


The penalties for not meeting the liabilities are very high.
The market value of liabilities is interest rate sensitive.
Risk taken in the investment portfolio limits the investor’s ability to profitably
take risk in other activities.
Tax incentives favor holding fixed-income securities.

(c) Compare the appropriateness of the traditional Sharpe Ratio measure and the Risk
Adjusted Change in Surplus (RACS) when managing the assets of this pension
plan.

APM Spring 2011 Solutions Page 35


16. Continued

The traditional Sharpe Ratio measure considers only the risk and return of assets,
ignoring the presence of any liability stream.

Some investment structures are better suited to hedge against the value of
liabilities than others. This hedging ability should be taken into account when
evaluating an investment, but is ignored when the Sharpe Ratio is used as an
evaluation measure.
In contrast, the RACS recognizes liabilities by considering the expected change
(mean) in surplus and the uncertainty (standard deviation) in the change in
surplus.
The RACS measures the dollar return on surplus that is in excess of the risk-free
rate of return against the risk taken relative to the risk-free strategy.
Hence RACS is more suitable when managing the assets of this pension plan.

(d)
(i) Calculate the RACS for each of Portfolio A and Portfolio B.

RACS(t)= Et[St+1– St (1+Rf)] / σt[St+1]


Where
σt[St+1] = σt[St+1- St (1+Rf)]

(ii) Recommend which of Portfolios A or B should be selected, based only on


the RACS.
Portfolio A:
St=A-L=110-100 10
E(St+1)=110*1.06-100*1.04 12.6
Et[St+1– St (1+Rf)]=12.6-10*1.03 2.3
σt[A]=110*12% 13.2
σt[L]=100*10% 10.0
σt2[St+1]=13.2^2+10^2-2*13.2*10*0.2 221.44
σt[St+1]=221.44^0.5 14.88
RACS(t)=2.3/14.88 15.46%
Portfolio B:
St=A-L=110-100 10
E(St+1)=110*1.05-100*1.04 11.5
Et[St+1– St (1+Rf)]=11.5-10*1.03 1.2
σt[A]=110*11% 12.1
σt2[St+1]=12.1^2+10^2-2*12.1*10*0.9 28.61
σt[St+1]=28.61^0.5 5.35
RACS(t)=1.2/5.35 22.43%

Since RACS(t)for Portfolio B > RACS(t) for Portfolio A, the manager


would choose Portfolio B.

APM Spring 2011 Solutions Page 36


16. Continued

(e) In addition to the information provided in (d), assume that the Pennywise Pension
Plan has a payout rate of 8% per year.

(i) Calculate the funded ratio for the Pension Plan at year-end 2009.

Funded Ratio, Ft = At/Lt


Portfolio B value in 2010 = 110
Pension Liabilities in 2010 = 100
Ft = 110/100 = 1.1

(ii) Calculate the minimum excess return over liabilities that the fund must
achieve in order to maintain the same funded ratio throughout 2010.

Et[Ft+1] = Ft Et[(1+RA,t+1)/ (1+RL,t+1)]* [1/(1-p)] - p/(1-p)

To maintain the funded ratio, assume Ft+1 = Ft

Et[(1+RA,t+1)/ (1+RL,t+1)] = [Et[Ft+1] (1-p) +p]/ Ft

Define Rx,t+1 = (1+RA,t+1)/ (1+RL,t+1)

Et[(Rx,,t+1)] = [Et[Ft+1] (1-p) +p]/ Ft


= [1.1 *(1- 0.08) + 0.08 ] / 1.1
= 0.9927
The minimum excess return over liabilities that the fund must achieve to
maintain a funded ratio of 1.39 over a one-year period is then 0.9927 – 1 =
-0.73%

(iii) Recommend a strategic asset allocation strategy, given that the Pension
Plan is overfunded, and justify your recommendation.

• The Pension Plan is overfunded with a funded ratio of 1.1.


• Since the Pension Plan is overfunded, the funded ratio of 1.1 can be
maintained even with a net negative asset over liabilities return.
• Overfunded pension plans can actually increase the probability of
losing surplus by allocating more to equities (since equities are
typically riskier).
• Recommend that the pension plan adopt a more conservative investing
strategy, for example by allocating more to fixed income type assets in
order to avoid the danger of falling below its target funded ratio.
• Other considerations that need to be taken into account:
o Expected future growth of pension liabilities
• Type of pension plan formula (consider salary growth, inflation,
expenses etc)
APM Spring 2011 Solutions Page 37
17. Learning Objectives:
1. Candidate will understand and be able to follow the investment management
process for insurance companies, pension funds and other financial
intermediaries.

2. The candidate will understand the variety of financial instruments available to


managed portfolios.

8. The candidate will understand the behavior characteristics of individual and firms
and be able to identify and apply concepts of behavioral finance.

Learning Outcomes:
(1a) Explain how an investment policy and an investment strategy can help manage
risk and create value.

(1b) Identify the obligations of a fiduciary in managing investment portfolios and


explain how they apply in a given situation.

(1c) Determine how a client’s objectives, needs and constraints affect the selection of
an investment strategy or the construction of a portfolio. Considerations include:
• Funding objective
• Risk-return trade-off
• Regulatory and rating agency requirements
• Risk appetite
• Liquidity constraints
• Capital, tax and accounting considerations

(2a) Compare and select specialized financial instruments that can be used in the
construction of an asset portfolio supporting financial institutions and pension
plan liabilities.

(8c) Identify and apply the concepts of behavioral finance with respect to investors,
option holders and policyholders, including optimal behavior, real behavior,
model behavior and empirical studies.

Sources:
Maginn & Tuttle, Chapter 3, Managing Institutional Investor Portfolios, pgs 65, 81-83

Fiduciary Liability Issues for Selection of Investments pgs 4-9

V-C146-09: The Role of Commodities in Investment Portfolios, pg 35

Byrne & Brools, Behavioral Finance: theory and Evidence, pgs 7-8

APM Spring 2011 Solutions Page 38


17. Continued

Commentary on Question:
This is a recall and evaluation question that is asking the candidates to (a) describe
purposes of establishing a SIP, (b) identify sponsor’s fiduciary responsibilities and (c)
identify and apply the concepts of behavioral finance, all in the context of a DC pension
plan.

Solution:
(a) Describe the purpose of establishing a Statement of Investment Policy in the
context of a DC pension plan.

The establishment of an IPS may be legally mandated.

The IPS documents the manner in which the plan sponsor is meeting the fiduciary
responsibility to have an adequate process for selecting the investment options
offered to play participants as well as for periodically evaluating those options. A
DC investment policy statement establishes procedures and governing principles
to ensure that a myriad of individual investor objectives and constraints can be
properly addressed.

In a DC plan, the plan sponsor does not establish objectives and constraints;
rather, the plan participants set their own risk and returns objectives and
constraints. The plan sponsor provides educational resources, but the participant
is responsible for choosing a risk and return objective reflecting his or her own
personal financial circumstances, and attitudes toward risk.

The primary focuses of an investment policy statement for a DC plan are as


follows:
• Clearly distinguish among the responsibilities of the committee, the plan
members, the fund managers, and plan trustee recordkeeper selected by the
committee.
• Provide descriptions of the investment alternatives available to plan
participants.
• Provide criteria for monitoring and evaluating the performance of investment
managers and investment funds relative to appropriate investment
benchmarks.
• Provide criteria for manager and fund selection, termination and replacement.
• Establish effective communication procedures for the fund managers, the
trustee recordkeeper, the committee, and the plan participants.

APM Spring 2011 Solutions Page 39


17. Continued

(b) Identify the responsibilities of ABC Company in developing the list of investment
options for a DC pension plan.

Duties of the trustee


1. Loyalty
2. Care
3. Diversify Plan Assets
4. Impartiality
5. Delegate
6. Follow Statutory Constraints
7. Make the Property Productive
8. Regarding Co-Trustees
9. Act in Accordance with the Trust Agreement

Diversification: The sponsor must offer a menu of investment options that allows
participants to construct suitable portfolios. Role is to provide participants with
an array of investment choices with various investment objectives and asset
classes with different risk and return characteristics that should enable participants
to invest according to their varying investment needs.
ERISA establishes a safe harbor for DC plan sponsors against claims of
insufficient or imprudent investment choice if the plan has at least 3 investment
choices diversified versus each other, and a provision for the participant to move
freely among options.
Other responsibilities include:
• Monitoring the funds’ investment performance relative to their objectives.
• Monitoring fees and ensuring they are reasonable
• Terminating and replacing funds when appropriate.
• Assuring ongoing communication with participants and appropriate education
resources.

(c) Describe the behavioral biases facing DC plan members.

Individual investors are overconfident.


Individual investors fail to behave rationally.
Individual investors use the 1/n diversification strategy.
Enthusiasm of participants to invest in the stock of their employer.
Naïve extrapolation of strong past performance.
Participant sticks with default option.
Investors are reluctant to realize losses and tend to sell winners and hold losers.
Investors react to short-term performance records.
Investors tend to reallocate their cash to funds that own stocks with low future
returns.

APM Spring 2011 Solutions Page 40


17. Continued

(d) Assess the appropriateness of each of the investment options suggested by the
committee member.

Money Market Fund has high liquidity, short-term, low return and risk with
sufficient diversification properties.

Large Cap Fund has high liquidity, higher return and risk compared to Money
Market Fund with sufficient diversification properties.

Venture Capital Fund seeks to earn a return in excess of public stock market.
There is business risk, liquidity risk and lack of diversification. Venture Capital
should expect large standard deviation of return. Low correlation with traditional
asset classes bonds and equities, effective diversifying asset class.

Commodity Fund provides diversification but with volatility higher than equities.
It may provide a good hedge on inflation.

Money Market Fund and Large Cap Fund are generally appropriate. Venture
Capital Fund and Commodity Fund may not be appropriate given their higher
risk, especially if only a limited number of options is offered.

(e) Recommend changes to the above list of investment options and justify your
recommendation.

The investment choices offered should represent asset classes with different risk
and return characteristics and with sufficient diversification properties.
Diversification: The sponsor must offer a menu of investment options that allows
participants to construct suitable portfolios.
Venture Capital Fund and Commodity Fund should be removed if only 4 funds
are offered.
Money Market could also be replaced.

Funds that could be considered:


• Balanced Fund
• Target Risk Fund
• Target Date Fund
• Life-Cycle Funds
• Fixed Income Fund
• Inflation-Indexed Fund
• Small Cap Equity Fund
• Foreign Equity Fund

APM Spring 2011 Solutions Page 41


17. Continued

(f) Recommend a default investment option for members who refrain from selecting
any options and justify your recommendation.

A product with a mix of investments that takes into account the individual’s age,
life expectancy, or target retirement date. An example of such a fund or portfolio
may be a “life-cycle” or “target-retirement date” fund.

A product with a mix of investments that takes into account the characteristics of
the employee group as a whole, rather than each individual (an example of such a
fund or portfolio may be a “balanced” fund).

A capital preservation product (such as a money market account) based on the


suggested options by the committee members, the Money Market Fund would be
the most appropriate default investment option.

APM Spring 2011 Solutions Page 42


7. (9 points) Company XYZ is considering the following two classes of fixed income
securities in which to invest the assets supporting their traditional life and fixed deferred
annuity insurance liabilities:

Investment Value Modified Basis Risk


Security ($M) Duration Adjustment Factor
Class AA 50 8 0.97
Class BB 50 8 0.92

Assume the following:

• Class AA is composed of bonds with sinking fund features

• Class BB is composed of callable bonds that are non-callable for the first 5 years
then callable with a make-whole feature

• Liabilities of $93M

• Effective liability duration of 8

(a) (1 point) Explain how bonds with sinking fund provisions work.

(b) (1 point) Describe the make-whole call feature and explain why it’s attractive to
borrowers and lenders.

One problem with the modified duration values shown in the table above is that each is
derived with respect to a different reference rate of interest.

(c) (1 point) List two benefits of having the effective duration of assets based on
changes in the same reference yield.

Exam APM – Fall 2012 -8- GO ONTO NEXT PAGE


Advanced Portfolio Management
Morning Session
7. Continued

Basis risk refers to the risk attributable to uncertain movements in the spread between
yields associated with a particular financial instrument or class of instruments and a
reference interest rate over time.

(d) (1 point) List some examples of basis risk.

(e) (3 points) Recommend a re-allocation of investments within the two available


asset classes to achieve surplus immunization.

(f) (2 points) Describe financial market conditions and XYZ insurance product
features that create an interest rate risk profile known as a “short-straddle”.

Exam APM – Fall 2012 -9- GO ONTO NEXT PAGE


Advanced Portfolio Management
Morning Session
15. (8 points) The Ocoee Teacher’s Pension Plan (OTPP) is a defined-benefit plan in
Canada. The plan is fully indexed to the Canadian Consumer Price Index (CPI). Due to
regulation changes, OTPP was tapped out of the Canadian real return bonds market. To
hedge its inflation risk, OTPP recently purchased a zero coupon U.S. Treasury Inflation
Protected Security (TIPS). You are given the following:

Trade date: July 17, 2011


Settlement date: July 22, 2011
Maturity date: December 31, 2021
Base index: U.S. CPI, January 2001
Day-count convention: 30/360
Number of days to maturity: 3,759
Nominal rate: 2.50%
Face value: $100,000,000
Settlement price: $115,000,000

U.S. Non-Seasonally Adjusted, All-Urban CPI (CPI-U) Series


Date Price
8/1/2011 226.268
7/1/2011 225.425
6/1/2011 217.235
5/1/2011 215.304
4/1/2011 214.118
1/1/2011 175.600

Note that the reference CPI-U for any valuation date incorporates a three-month lag.

(a) (1 point) List the tactical and strategic advantages of TIPS for investors.

(b) (3 points) Calculate the break-even inflation rate priced into the zero coupon
TIPS.

(c) (1 point) Outline the challenges and concerns this inflation hedging strategy
might face.

(d) (1 point) Assess how the inflation hedging strategy would perform if both U.S.
and Canada experience deflation over a decade.

(e) (2 points) Describe the advantages and disadvantages of investing in each of the
following asset classes to hedge against unexpected changes in inflation.

(i) Infrastructure

(ii) Timberland

Exam APM – Fall 2012 -9- GO ONTO NEXT PAGE


Advanced Portfolio Management
Afternoon Session
7. Learning Objectives:
2. The candidate will understand the variety of financial instruments available to
managed portfolios.

4. The candidate will understand and apply quantitative techniques for portfolio
management.

Learning Outcomes:
(2a) Compare and select specialized financial instruments that can be used in the
construction of an asset portfolio supporting financial institutions and pension
plan liabilities.

(4c) Describe, contrast and assess techniques to measure interest rate risk.

Sources:
Fabozzi, Handbook of Fixed Income Securities, 7th Edition, 2005

Default Risk and the Effective Duration of Bonds

Ineffective Duration Measures for Life Insurers

Commentary on Question:
The purpose of this question is to test whether the candidate can describe various bond
features and sources of basis risk between bonds. This is tested via recall type questions
as well as computational questions requiring the use of a basis risk adjustment factor to
calculate the effective duration of surplus. Basic calculations using effective durations
are also tested.

Solution:
(a) Explain how bonds with sinking fund provisions work.

The sinking fund features require the issuer to liquidate all or portion of their
bond issues periodically before maturity. Usually, the sinking-fund call price is
the par value if the bonds were originally sold at par. When issued at a price in
excess of par, the sinking-fund call price generally starts at the issuance price and
scales down to par as the issue approaches maturity. May have a deferment
period which prevents the feature from being exercised within the first few years.
There are two ways to retire the bonds:
1. The issuer pays cash of the face amount of the bonds to the trustee, who then
calls the bonds by lottery for redemption, or
2. The issuer retires the bonds in the open market if the price is below par ,
although few corporate bond indentures may prohibit the open-market
purchase of the bond by the issuer.

(b) Describe the make-whole call feature and explain why it’s attractive to borrowers
and lenders.

APM Fall 2012 Solutions Page 20


7. Continued

Firm can buy back the bond issues prior to maturity at the call price that is the
higher of the following:
1. Par value plus interest
2. Present value of remaining coupons and principal, discounted at a Treasury
rate that matches with the bond’s remaining maturity plus a spread specified
in the indenture
The cost for the issuer is lower. The make-whole call price increases as the
interest rate decreases, therefore the issuer will have less incentive to exercise the
call simply because the interest rate is declining. Therefore, the bondholder has
more protection and thus demand lower return

(c) List two benefits of having the effective duration of assets based on changes in the
same reference yield.

 Easier to compare the interest rate sensitivities of different securities


 Isolate the impact of credit risk
 Compute portfolio duration when multiple types of securities are present

(d) List some examples of basis risk.

 Long-term and short-term financial instruments


 Domestic currency and foreign currency
 Liquid and illiquid investments
 Bonds with high versus low sensitivity to changes in interest rate volatility
 Taxable and tax-free instruments
 Spot and futures contracts
 Default free and non-default free securities

(e) Recommend a re-allocation of investments within the two available asset classes
to achieve surplus immunization.

Step 1 – Calculate the Duration of Assets such that Ds = 0


DS = (DA − DL) * (A/S) + DL
Immunization is achieved when Duration of Surplus, Ds, equals 0.
Rewrite the equation for DS in terms of DA; DA = (DS − DL) * (S/A) + DL
Setting DS = 0; DA = (0 − 8) * (7 / 100) + 8 = 7.44

APM Fall 2012 Solutions Page 21


7. Continued

Step 2 - Solve for the AA% that achieves the target portfolio duration of 7.44
using the individual assets’ effective durations:
7.44 = AA% * 8 * .97 + (1 − AA%) * 8 * .92
AA% = (7.44 – 8 * .92) / (8 * .97 – 8 * .92) = 20%
Thus, Class AA allocation = $20M, Class BB Allocation = $80M

(f) Describe financial market conditions and XYZ insurance product features that
create an interest rate risk profile known as a “short-straddle.”

Asset displays negative convexity due to prepayment and call provisions.


Liability displays positive convexity due to:
 Guarantees on credited rates
 Book value withdrawal (i.e. receive AV - surrender charge from insurance
company)
Short straddle position means the insurance company takes a loss regardless of
which direction the interest rate moves.
 When interest rate increases, PH lapse more to take advantage of book value
withdrawal and seek other more attractive investment options, the
depreciation in assets supporting liability could be more than the surrender
charge, hence insurance company takes a loss.
 When interest rate decreases, PH lapse less to take advantage of minimum
guaranteed credit rate, the assets tend to get called by the issuer, which leaves
the insurer with large sums of cash to reinvest in a low interest rate
environment, hence insurance company takes a loss.

APM Fall 2012 Solutions Page 22


15. Learning Objectives:
2. The candidate will understand the variety of financial instruments available to
managed portfolios.

Learning Outcomes:
(2a) Compare and select specialized financial instruments that can be used in the
construction of an asset portfolio supporting financial institutions and pension
plan liabilities.

Sources:
John Brynjolfsson, Handbook of Fixed Income Securities, 7th Edition, 2005, Chapter 15
Inflation-Linked Bonds, pgs. 353 – 357, 368 and 372

V-C154-09: Robert G. Bertram, Balancing the Opportunities in Real Return Investments,


2007, pgs 46 – 47 and 49

Commentary on Question:
This question is testing the candidates’ understanding of TIPS. It is also testing the
knowledge of other asset classes that could hedge against unexpected changes in
inflation.

Solution:
(a) List the tactical and strategic advantages of TIPS for investors.

Tactical Advantages
Investor can speculate using TIPS on changes in inflation and real interest rates.

Strategic Advantages
 TIPS can enable the investors to obtain a high real yield.
 Diversification: TIPS have a low correlation to traditional financial assets.
 TIPS have muted volatility.

(b) Calculate the break-even inflation rate priced into the zero coupon TIPS.

CPI-U for any valuation date incorporates a 3-month lag

Calculate Index Factor

APM Fall 2012 Solutions Page 42


15. Continued

Calculate TIPS real yield

TIPS real yield = 0.6%

Calculate breakeven inflation rate

(c) Outline the challenges and concerns this inflation hedging strategy might face.

Two key concerns:


 Inflation mismatch – TIPS are based on the U.S. CPI, which may not
necessarily grow at the same rate as the Canadian CPI. This introduces basis
risk.
 Currency mismatch – The strategy is exposed to exchange rate risk. Investing
in TIPS creates US dollar exposure, which may necessitate the need for
Canadian/US currency hedging thus incurring additional costs.

(d) Assess how the inflation hedging strategy would perform if both U.S. and Canada
experience deflation over a decade.

The inflation hedging strategy should perform well if both the U.S. and Canada
experience deflation over a decade.

APM Fall 2012 Solutions Page 43


15. Continued

Under a prolonged deflationary environment, the plan liabilities will decrease


since the benefits are indexed to the Canadian CPI, but TIPS are guaranteed to at
least pay out the original principal payment. This floor allows the hedge to be
beneficial when both countries experience prolonged deflation.

(e) Describe the advantages and disadvantages of investing in each of the following
asset classes to hedge against unexpected changes in inflation.

(i) Infrastructure

(ii) Timberland

Infrastructure
Advantages
 Earn stable income while unexpected inflation hedged
 Low correlation with traditional assets
Disadvantages
 Illiquid
 High maintenance and due diligence costs
 Increasing investor awareness – so cost of asset is increasing and prospective
returns are declining

Timberland
Advantages
 Returns are largely dependent on organic growth
 Low correlation with traditional assets
Disadvantages
 Illiquid
 Neighborhood deterioration risk (e.g. fires)
 Increasing investor awareness – so cost of asset is increasing and prospective
returns are declining

APM Fall 2012 Solutions Page 44


5. (7 points) ABC Life splits a 5-year fixed-rate bond with a market value equal to par
value of $100 million, and a yield of 5.5% into a floater and an inverse floater with par
value of $80 million and $20 million, respectively. The coupon rate for the floater will
equal the 3-month LIBOR. The inverse floater has neither a cap rate nor a floor rate.

(a) (1 point) Determine the coupon formula for the inverse floater.

(b) (2 points) Determine whether the duration of the inverse floater is higher or lower
than that of the original fixed-rate bond and explain why.

(c) (2 points) Describe a situation where it would be appropriate to include an


inverse floater in the asset portfolio to manage the interest rate risk of a fixed
payout liability portfolio.

(d) (2 points) Your manager suggested using the “discount margin” method to
evaluate a callable floater.

(i) Explain how “discount margin” method works.

(ii) Assess the manager’s suggestion.

Exam APM – Spring 2013 -7- GO ONTO NEXT PAGE


Advanced Portfolio Management
Morning Session
7. (6 points) You are an actuary working for a life insurance company heavily invested in
structured securities.

Your manager understands that prepayments are the primary distinguishing feature of
Agency Mortgage Backed Securities, but does not know what factors will impact
prepayment behavior.

(a) (1.5 points) List and describe key factors that impact prepayment behavior.

He came across an article that discusses different prepayment conventions. He wants to


learn more, and asks you to:

(b) (1.5 points) Describe the following conventions:

(i) Single Monthly Mortality

(ii) Conditional Prepayment Rate

(iii) Public Securities Association Model

(c) (1.5 points) Compute the prepayment rate using above conventions, assuming:
• Scheduled Balance: $150,000
• Actual Balance: $148,000
• Age (months): 24

(d) (1.5 points) Your manager told you that AAA rated CDOs are far riskier than
AAA rated corporate bonds.

Explain the rationale behind his assertion.

Exam APM – Spring 2013 -9- GO ONTO NEXT PAGE


Advanced Portfolio Management
Morning Session
9. (4 points)

(a) (1.5 points) List and describe the duties of a trustee as they relate to tax-favored
retirement plans.

(b) (0.5 points) Explain the difference in the prudent investor standard between
ordinary trusts and retirement plan trusts.

(c) (2 points) For a defined-contribution retirement plan with self-directed accounts,


the following investment choices are available to plan participants:

• Foreign Stock Fund


• Short-term Government Bond Fund
• Gold Mining Sector Equity Fund
• Commercial Real-Estate Investment Trust
• Company Stock of Employer

(i) Assess the appropriateness of the offerings and recommend any changes.

(ii) Compare the legal liability of the trustee and plan participants as it relates
to investment selection and performance.

**END OF EXAMINATION**
Morning Session

Exam APM – Spring 2013 - 11 - STOP


Advanced Portfolio Management
Morning Session
USE THIS PAGE FOR YOUR SCRATCH WORK
11. (7 points) You are a consultant hired by insurance company XYZ to evaluate their ALM
process. During your review, you find the following:

• Company XYZ’s assets include treasuries, corporate bonds, commercial


mortgages and MBS

• XYZ’s goal is to match the duration of assets and liabilities

• They calculate and report effective duration of liability and assets quarterly.
Below are the results from the ALM model.

Interest Rate Move Assets Liability


Shock Up 50 bps 180 M 160 M
No shocks 200 M 180 M
Shock Down 50 bps 210 M 220 M

(a) (1 point) Compare Macaulay duration, empirical duration, effective duration and
key rate duration.

(b) (1 point) Explain the possible reasons that the value of assets could move
differently from the duration predicted value.

(c) (1 point) Identify risks that XYZ faces in a falling interest rate environment.

(d) (2.5 points) Estimate the value of assets and liabilities following a 70bps drop in
interest rates using duration and convexity.

(e) (1.5 points) Evaluate suitability of adding the following assets to improve the
company’s asset liability risk management.

(i) 30 year high yield bond

(ii) 15 year callable investment grade bond

(iii) 5 year government bond

Exam APM – Spring 2013 -2- GO ONTO NEXT PAGE


Advanced Portfolio Management
Afternoon Session
12. (7 points) Tom is the portfolio manager for ABC Life’s investments in agency MBS,
agency CMOs and mortgage bonds.

(a) (1 point) Describe each of the assets: agency MBS, agency CMOs and mortgage
bonds.

(b) (1 point) Compare the credit risks of each instrument.

(c) (1 point) Compare the prepayment risks of each instrument.

(d) (2 points) As part of the company’s ALM interest rate risk policy, Tom is
required to manage his portfolio to match a target duration and cash-flow profile
to fund a specific set of liabilities. ABC Life is looking to better measure Tom’s
performance by implementing a benchmark for his agency MBS investments.

(i) Describe the considerations in selecting a benchmark.

(i) Recommend a benchmark for the agency MBS portfolio.

(e) (1 point) ABC Life is also looking to setup an index for Tom’s mortgage bond
portfolio to mitigate risk.

Identify the key risk associated with a fixed income credit portfolio and explain
ways to mitigate this risk.

(f) (1 point) Tom plans to use a stratified sampling approach to replicate the credit
index.

Explain whether this is an appropriate approach.

Exam APM – Spring 2013 -3- GO ONTO NEXT PAGE


Advanced Portfolio Management
Afternoon Session
14. (6 points) ABC Life is considering issuing corporate bonds, but wants to include
mechanisms in the bond’s indenture that allow the Company to retire the debt before
maturity.

(a) (1 point) List and describe the mechanisms that ABC can use.

(b) (2 points) Four years ago ABC issued $10 million of 10-Year bonds with 6%
coupon paying annually. These bonds are redeemable at any time in whole or in
part at ABC’s option. The make-whole call premium is 10 basis points. Current
CMT rates are shown in the table below.

Maturity
1 2 3 4 5 6 7 8 9 10
(years)
Yield 0.50% 0.80% 1.10% 1.40% 1.70% 2.00% 2.30% 2.60% 2.90% 3.20%

Calculate the redemption price.

(c) (1 point) Compare the convexity of a bond with the fixed-priced call provision
relative to a bond with the make-whole call provision.

(d) (1 point) Recommend a provision to be included in the bond’s indenture that


helps mitigate the credit risk for ABC’s bondholders.

(e) (1 point) ABC’s CFO believes that an extendible reset bond is the same as a
floating-rate asset because the coupon rate may reset annually or even more
frequently.

Critique this statement.

Exam APM – Spring 2013 -5- GO ONTO NEXT PAGE


Advanced Portfolio Management
Afternoon Session
5. Learning Objectives:
2. The candidate will understand the variety of financial instruments available to
managed portfolios.

6. The candidate will understand the specific considerations relative to managing a


fixed income portfolio within an asset allocation framework.

Learning Outcomes:
(2a) Compare and select specialized financial instruments that can be used in the
construction of an asset portfolio supporting financial institutions and pension
plan liabilities.

(6d) Recommend an investment strategy for a given situation:


• Portfolio policy and objectives
• Asset selection criteria
• Capital market expectations
• Risk management strategy
• Portfolio rebalancing strategy

Sources:
Fabozzi, Frank, The Handbook of Fixed Income Securities, 8th Edition, Chapter 17

Commentary on Question:
Parts (a), (b) and (c) of the question tested candidates’ knowledge on the characteristic of
floater and inverse floater. Part (d) tested candidates’ knowledge on “Discount Margin”
method, and ability to point out why “Discount Margin” is not a good method to evaluate
assets with embedded options.

Solution:
(a) Determine the coupon formula for the inverse floater.

Coupon payment on the collateral = Coupon payment on floater + Coupon


payment on Inverse floater

Coupon payment on the collateral = NxC = 100x5.5% = 5.5


Coupon payment on floater = 0.8NxLIBOR = (0.8x100)xLIBOR = 80xLIBOR
Coupon payment on Inverse floater = Coupon payment on the collateral – Coupon
payment on floater
= 5.5-80xLIBOR
Coupon rate on the Inverse floater = Coupon payment on Inverse floater/Par value
= (5.5-80xLIBOR)/20
=27.5%-4xLIBOR

(b) Determine whether the duration of the inverse floater is higher or lower than that
of the original fixed-rate bond and explain why.

APM Spring 2013 Solutions Page 18


5. Continued

• Duration of the Inverse floater will be higher than the original fixed-rate bond
• The dollar duration of the collateral must equal the dollar duration for the
combined floater and inverse floater
• Floaters have low duration because floaters adjust to the interest rate
periodically
• The payments on the inverse floater are leveraged and will have a higher
duration

(c) Describe a situation where it would be appropriate to include an inverse floater in


the asset portfolio to manage the interest rate risk of a fixed payout liability
portfolio.

• Inverse floater may be a useful hedge against reinvestment risk as the coupon
payments increase when interest rates decrease.
• Inverse floater has high duration and may be a suitable instrument in
extending the overall duration of the portfolio for ALM matching.

(d) Your manager suggested using the “discount margin” method to evaluate a
callable floater.

(i) Explain how “discount margin” method works.

• Discount margin method determines the average margin over the


reference rate the investor can earn over the security’s life given a
particular assumed path that the reference rate will take to maturity.
• Process:
o Determine the cash flows given a particular reference rate
o Set a margin
o Discount the cash flows using reference rate plus the margin
selected in prior step
o Repeat the process until the present value of the cash flows equal
to the price. Discount margin is the margin over the reference rate
such that the present value of the cash flows equal to the price.

(ii) Assess the manager’s suggestion.

• Not a good idea to use “Discount Margin” method


• “Discount Margin” method does not recognize the presence of
embedded options

APM Spring 2013 Solutions Page 19


5. Continued

• Since issuer can call an issue when presented with the opportunity and
refund at a lower spread. Investor will then reinvest at the lower
spread. Hence, it is important to recognize the embedded options and
value them properly.
• Can value the call option using
o Arbitrage-free binomial interest rate trees
o Monte Carlos simulations

APM Spring 2013 Solutions Page 20


7. Learning Objectives:
2. The candidate will understand the variety of financial instruments available to
managed portfolios.

Learning Outcomes:
(2a) Compare and select specialized financial instruments that can be used in the
construction of an asset portfolio supporting financial institutions and pension
plan liabilities.

Sources:
Fabozzi, Handbook of Fixed income Securities, 8th Edition, 2012
• Chapter 25, Agency Mortgage-Backed Securities

V-C174-09: Anson, The Handbook of Alternative Assets, Second Edition, 2006, Chapter
20

V-C179-10: The Economics of Structured Finance, Journal of Economic Perspective,


Winter 2009

Commentary on Question:
None

Solution:
(a) List and describe key factors that impact prepayment behavior.

Level of interest rates – reflects a borrower’s opportunity to refinance.


Prepayment speed increases when cost of refinancing decreases.

Aging – reflects that new loans typically exhibit slower prepayment speeds
compared with “seasoned” loans.

Loan - Premium loans experience higher prepayment speeds sooner compared


with the discount loan.

Burnout – Refinancing activity within a loan pool declines over time regardless
of interest rate environment.

Seasonality – reflects the close interaction of prepayments with housing maket


activity. Prepayments tend to be faster during the summer and slower during
winter months, reflecting increased home turnover during the summer months.
Houses prices and general economy can affect prepayments. Weak economic
activity, declining housing prices and unemployment depress prepayments.

Shape of the yield curve – when the yield curve is steep, borrowers may
refinance into shorter maturity loans to reduce the borrowing costs.

APM Spring 2013 Solutions Page 24


7. Continued

(b) Describe the following conventions:

(i) Single Monthly Mortality

(ii) Conditional Prepayment Rate

(iii) Public Securities Association Model

Single Monthly Mortality (SSM) – measures the percentage of dollars prepaid in


any month, expressed as a percentage of the expected mortgage balance.

Conditional Prepayment Rate (CPR) – converts SSM to an annual rate.


Expressed as a percentage of nonamortized balance prepaid on an annual basis.

Public Securities Association Model – expressed in CPR, where prepayment


rates are assumed to follow a standard path over time.

(c) Compute the prepayment rate using above conventions, assuming:


• Scheduled Balance: $150,000
• Actual Balance: $148,000
• Age (months): 24

SSM = 100 x (Scheduled Balance – Actual Balance)/Scheduled Balance


SSM = 100 x (150000-148000)/150000
SSM = 1.33%
CPR = 100 x [1-(1-SMM/100)^12]
CPR = 100 x [1-(1-1.33/100)^12]
CPR = 14.88%
PSA = 100x CPR/Minimum (age,30)x0.2
PSA = 100x14.88/Min(24,30)x0.2
PSA=309.94%

(d) Your manager told you that AAA rated CDOs are far riskier than AAA rated
corporate bonds.

Explain the rationale behind his assertion.

APM Spring 2013 Solutions Page 25


7. Continued

Modest imprecision in the parameter estimates can lead to variation in the default
risk that is sufficient to cause a security rated AAA to default with reasonable
likelihood.

The pooling of tranching amplified mistakes in the assessment of underlying asset


defaul risk and correlation.
It substitutes risks that are largely diversifiable for risks that are highly
systematic.
Therefore, structured securities have far less chance of surviving a severe
economic downturn than traditional corporate securities of equal rating.

APM Spring 2013 Solutions Page 26


9. Learning Objectives:
1. Candidate will understand and be able to follow the investment management
process for insurance companies, pension funds and other financial
intermediaries.

Learning Outcomes:
(1b) Identify the obligations of a fiduciary in managing investment portfolios and
explain how they apply in a given situation.

Sources:
V-C136-10: Fiduciary Liability Issues for Selection of Investments

Maginn & Tuttle, Managing Investment Portfolios, 3rd Edition, 2007


• Chapter 3, Managing Institutional Investor Portfolios, by Tschampion, Siegel,
Takahashi & Maginn

Commentary on Question:
Commentary listed underneath question component when available.

Solution:
(a) List and describe the duties of a trustee as they relate to tax-favored retirement
plans.

1. Duty of Loyalty
For the benefit of the beneficiaries
2. Duty of Care
Manage with attention and skill
3. Duty to Diversify Plan Assets
Minimize the risk of large losses
4. Duty of Impartiality
Does not favor one beneficiary at the expense of another
5. Duty to Delegate
Can delegate authority to investment manager, but retains responsibility
6. Duty to Follow Statutory Constraints
Usually involve self-dealing and imprudent investments
7. Duty to Make the Property Productive
Seek a reasonable return on the investment
8. Duties Regarding Co-Trustees
Cooperate with, keep an eye on co-trustees
9. Duty to Act in Accordance with the Trust Agreements
Should not breach other duties, to the detriment of plan participants

APM Spring 2013 Solutions Page 29


9. Continued

(b) Explain the difference in the prudent investor standard between ordinary trusts
and retirement plan trusts.

With retirement plans, duty is higher than with ordinary trusts


Duty to act as “professional investor” not just prudent person
ERISA defines in the U.S.

(c) For a defined-contribution retirement plan with self-directed accounts, the


following investment choices are available to plan participants:

• Foreign Stock Fund


• Short-term Government Bond Fund
• Gold Mining Sector Equity Fund
• Commercial Real-Estate Investment Trust
• Company Stock of Employer

(i) Assess the appropriateness of the offerings and recommend any changes.

(ii) Compare the legal liability of the trustee and plan participants as it relates
to investment selection and performance.

Commentary on Question:
For part (i) candidates were expected to point out the choices that may not be
appropriate and why, and also make suggestions to round out the options
available to plan participants. Reasonable answers all received credit. The
example below would have received full credit.
.

(i)
ERISA requires at least 3 “core” investment options
Company stock should be limited
Sector funds may be too risky
Foreign stock fund, ST Govt bond fund, REIT all appropriate
Consider adding:
• Domestic equity fund
• Corporate bond fund
• TIPS
• Money market fund
• Lifecycle funds

APM Spring 2013 Solutions Page 30


9. Continued

(ii)
The trustee remains legally responsible for investment decisions, until
responsibility is successfully shifted to the participants.
If responsibility is shifted, trustee not liable for duty of care with respect to
specific decisions made by participants.
Shifting responsibility defined as (by DOL):
1. At least 3 core options
2. Ability to transfer with appropriate frequency
3. Sufficient info to make informed decisions

APM Spring 2013 Solutions Page 31


11. Learning Objectives:
4. The candidate will understand and apply quantitative techniques for portfolio
management.

Learning Outcomes:
(4c) Describe, contrast and assess techniques to measure interest rate risk.

Sources:
V-C191-11: B. Tuckman, Fixed Income Securities, Chapters 5 -7

Commentary on Question:
The question addressed the topic of duration and duration matching. The question was
relatively easy and the candidates’ performance was above average, with many
candidates able to show a good understanding of these concepts.

Solution:
(a) Compare Macaulay duration, empirical duration, effective duration and key rate
duration.

Macaulay duration is the time-weighted present value of the cash flows divided
by price. It measures the price change relative to the yield-to-maturity change and
does not account for cash flows changing by interest rate scenario.

Effective duration measures price sensitivity to parallel treasury rate changes


assuming no change in other factors. It can account for cash flows changing by
interest rate scenario.

The key rates duration technique uses several key rates to define the term
structure of whole yield curve. The price sensitivity to the movement of each key
rate, holding the other constant, is the key rate duration. The sum of all key rate
durations will be similar to effective duration.

Empirical duration is obtained by regressing daily price changes against


corresponding yield changes using historical data.

(b) Explain the possible reasons that the value of assets could move differently from
the duration predicted value.

Duration predicts the change in value for small, parallel changes in the yield
curve. The following could cause the value of assets to move differently from the
duration predicted value:

• Convexity may be high. Duration itself could therefore be changing as


interest rates move.
• The yield curve may have a non-parallel deformation. For example, a change
in slope or curvature, which is not captured by effective duration

APM Spring 2013 Solutions Page 36


11. Continued

• Credit spreads may be changing.


• Price movements may be asymmetric due to optionality in the assets as
interest rates move up and down.

(c) Identify risks that XYZ faces in a falling interest rate environment.

The company’s asset duration is shorter than liability duration and in a falling
interest rate environment it will not have enough assets to cover liabilities.

In addition, the company faces pre-payment and call risk in falling interest rate
environment and this exacerbates reinvestment risk, where the company is forced
to reinvest at lower rates. The reinvested assets may not earn enough yield to
support liability products which are priced at higher interest rates.

Furthermore, certain options/guarantees on the liability side may move into-the-


money on the liability side when interest rates fall, which could further squeeze
profitability and lower surplus.

(d) Estimate the value of assets and liabilities following a 70bps drop in interest rates
using duration and convexity.

Commentary on Question:
While a correct solution would receive full credit, it is important to list out the
steps/formulas for partial credit in case any calculation errors occur.

Duration = - ( P+-P-)/(2*P*∆y)
Convexity = (P+ + P- - 2P) / (P*∆y2)
Duration of asset = - (180-210)/ (2*200*0.005) = 15
Duration of liability = -(160-220)/(2*180*0.005) = 33
Convexity of asset = (180+210-2*200)/(200 * 0.0052)= -2000
Convexity of liability =(160+220 – 2*180)/(180 * 0.0052)= 4444

When rates decrease 70 bps:

Asset price = 200 * (1 – (15*-0.007) + ½ * -2000 * (-0.007) 2)


= 200+21-9.8 = 211.2 M

Liability price = 180 * (1 – (33 *-0.007) + ½ * 4444 * (-0.007) 2)


=180 +41.6+19.6=241.2 M

APM Spring 2013 Solutions Page 37


11. Continued

(e) Evaluate suitability of adding the following assets to improve the company’s asset
liability risk management.

(i) 30 year high yield bond

(ii) 15 year callable investment grade bond

(iii) 5 year government bond

The company currently has a negative duration gap, with asset duration < liability
duration. To improve the company’s asset liability risk management, we would
prefer assets that could extend duration, which have positive convexity.

(i) 30-year high yield bond would extend the duration of asset portfolio, but it
would add credit risk. While high coupons may lower duration, it should
still be able to extend the duration of the portfolio.

(ii) Callable bond has negative convexity which is undesirable vis-à-vis the
liabilities. In addition, the 15-year callable bond will not be able to extend
asset duration (currently at 15) since the duration of a 15-year callable will
be less than 15 yr. Overall, this asset is not recommended.

(iii) The 5-year government bond won’t help alleviate the duration gap since it
has too short a duration and will not lengthen the asset portfolio duration.
Therefore, this asset is also not recommended

Of the three assets, the 30-year high yield bond would be the most suitable for
improving the ALM profile, despite the added credit risk.

APM Spring 2013 Solutions Page 38


12. Learning Objectives:
2. The candidate will understand the variety of financial instruments available to
managed portfolios.

7. The candidate will understand the purposes and methods of portfolio performance
measurement.

Learning Outcomes:
(2a) Compare and select specialized financial instruments that can be used in the
construction of an asset portfolio supporting financial institutions and pension
plan liabilities.

(7b) Describe and assess techniques that can be used to select or build a benchmark for
a given portfolio or portfolio management style.

(7c) Recommend a benchmark for a given portfolio or portfolio management style.

Sources:
Fabozzi, Handbook of Fixed Income Securities, 8th Edition, 2012
• Chapter 12, Corporate Bonds
• Chapter 24, An Overview of Mortgages and the Mortgage Market
• Chapter 25, Agency Mortgage-Backed Securities
• Chapter 26, Agency Collateralized Mortgage Obligations
• Chapter 50, Quantitative management of Benchmarked Portfolios

Commentary on Question:
Candidates needed to understand different financial instruments and assess their risk.
For parts (a), (b) and (c), candidates did well to explain an Agency MBS or CMO but
most did not understand what a mortgage bond was.

Solution:
(a) Describe each of the assets: agency MBS, agency CMOs and mortgage bonds.

Agency Mortgage backed securities (MBS)


• Cash flows of an underlying pool of mortgages – mortgage pass through
• Securitized by a government agency such as Ginnie Mae, or a government-
sponsored enterprise (GSE) such as Fannie Mae or Freddie Mac
• Non-conforming (“Jumbo”) loans do not qualify for the agency loan program

Agency Collateralized Mortgage Obligations (CMO)


• Underlying collateral is the same as MBS (loans on residential)
• CMO’s are structured so that there are several classes of bondholders with
varying maturities
• Principal is divided up into pieces / tranches

APM Spring 2013 Solutions Page 39


12. Continued

Mortage bonds
• Bondholder is granted a first- mortgage lien on all properties of the issuer
• A lien is a legal right to sell mortgaged property to satisfy unpaid obligations
to bondholders

(b) Compare the credit risks of each instrument.

Agency Mortgage backed securities (MBS)


• Very limited credit risk
• Ginnie Mae guarantee (full faith of the US) – explicit guarantee
• Fannie Mae and Freddie Mac, not backed by full faith of the US but have
access to support from the U.S. Treasury – implicit guarantee

Agency Collateralized Mortgage Obligations (CMO)


• Same situation as agency MBS – very limited credit risk

Mortgage bonds
• Very limited credit risk (granted a first-mortgage lien on all properties)

(c) Compare the prepayment risks of each instrument.

Agency Mortgage backed securities (MBS)


• Very high prepayment risk
• Generally no penalties on prepayments on residential mortgage loans
• Cash flows are greatly affected by the prepayment behavior of underlying
mortgages within the MBS pool

Agency Collateralized Mortgage Obligations (CMO)


• Very dependent on the tranche/structure
• Some tranches allows investors more control when they receive principal from
MBS
• In general, Sequentials tranches are very sensitive to prepayments
(particularly longer-duration sequentials)
• PAC is the most stable tranche of CMO
• Companion tranche absorb whatever principal is left

Mortgage bonds
• Limited prepayment risk
• Non-callable or callable but with punitive covenants (e.g. make-whole
clause): difficult to retire debt before maturity

APM Spring 2013 Solutions Page 40


12. Continued

(d) As part of the company’s ALM interest rate risk policy, Tom is required to
manage his portfolio to match a target duration and cash-flow profile to fund a
specific set of liabilities. ABC Life is looking to better measure Tom’s
performance by implementing a benchmark for his agency MBS investments.

(i) Describe the considerations in selecting a benchmark.

(ii) Recommend a benchmark for the agency MBS portfolio.

Commentary on Question:
Most candidates scored well on the considerations for benchmark selection,
however candidates did not convey a good understanding of the challenges in
benchmarking an MBS portfolio that is managed to a stable duration target.

(i) A benchmark should (1) be diversified (2) reflect the investor’s


opportunity set and consider any constraints such as required allocations
or duration matching criteria (3) be investable so that a manager could
“buy the benchmark.”

(ii) Recommend a custom, “liability-based” benchmark to match the duration


and cash-flow profile of the liabilities. However, the duration of an MBS
portfolio can be quite volatile. Therefore in order to maintain a stable
duration the manager may engage in delta hedging to overcome the impact
of negative convexity. The benchmark should reflect the cost of this delta
hedging in two ways (1) apply delta hedging to the benchmark or (2)
construct a “constant duration” index that provides a fairer benchmark for
a delta-hedged mortgage portfolio.

(e) ABC Life is also looking to setup an index for Tom’s mortgage bond portfolio to
mitigate risk.

Identify the key risk associated with a fixed income credit portfolio and explain
ways to mitigate this risk.

Excessive exposure to individual issuers is the key risk in a credit index. Two
possible ways to mitigate this risk are (1) using an issuer-capped index whereby a
cap is placed on the largest issuers to limit exposure (2) use a swap-based
benchmark as a total return benchmark for the credit portfolio due to the close
relationship between interest rate swap spreads and high-grade credit spread. The
added benefit of a swap-based benchmark is it alleviates the pressure for a
manager to have at least some exposure to the largest issuers in the benchmark,
especially in cases where the manager has a negative view on the issuer.

APM Spring 2013 Solutions Page 41


12. Continued

(f) Tom plans to use a stratified sampling approach to replicate the credit index.

Explain whether this is an appropriate approach.

Stratified sampling is a form of cash replication that attempts to match an index’s


allocation to each important segment with a few securities. It works well in
homoeneous markets, such as US governments or MBS. It works less well for
markets with much idiosyncratic risk such as credit so this is not an appropriate
approach to construct a proxy credit portfolio. Matching broad risk dimensions
still leaves the proxy portfolio vulnerable to issuer-level risk because the proxy
over-weights each issuer relative to the benchmark. It also ignores historical
variances and correlations of risk factors.

APM Spring 2013 Solutions Page 42


14. Learning Objectives:
4. The candidate will understand and apply quantitative techniques for portfolio
management.

Learning Outcomes:
(4e) Describe, contrast and assess techniques to measure liquidity risk.

Sources:
Fabozzi, Handbook of Fixed Income Securities, 8th Edition, 2012
• Chapter 12, Corporate Bonds

Commentary on Question:
Commentary listed underneath question component.

Solution:
(a) List and describe the mechanisms that ABC can use.

Commentary on Question:
Most candidates only mentioned call and sinking fund options.

Four options can be used:


• Call and refunding provision, which enables the issuer to buy bonds back
at a fixed price or market price
• Sinking-fund provision can be used to retire a portion of the debt each
year
• Maintenance and replacement funds used to retire debt
• Sale of assets and/or collateral
(b) Calculate the redemption price.

Commentary on Question:
Most candidates did not get the yield right, or state redemption price is the larger
of the present value and principal and accrued interest.

Redemption price is the larger of principal and accrued interest, and the make-
whole redemption price.
• Make-whole redemption price is the present value of remaining coupon
and principal, valued with treasury rate plus call premium =6-year CMT +
call premium= 2% + 0.1%= 2.1%
• Coupon = 60,000 per year (6 years), principal=10m at year 6
• Make-whole redemption price= PV(60,000) for 6 years + PV(10 M) at
year 6 at 2.1% =12,177,205
• Principal + accrued interest = 10M + 60k=10.6M
• Redemption price = max(12M, 10.6M)=12M.

APM Spring 2013 Solutions Page 46


14. Continued

(c) Compare the convexity of a bond with the fixed-priced call provision relative to a
bond with the make-whole call provision.

Commentary on Question:
Most candidates did not state make-whole convexity is positive.

Fixed price call provisions have negative convexity; make-whole call provisions
have positive convexity.

(d) Recommend a provision to be included in the bond’s indenture that helps mitigate
the credit risk for ABC’s bondholders.

Commentary on Question:
Most candidates only mentioned sinking-fund provision.

• Grant bondholders lien on properties


• Use collateral
• Limit additional debt
• Include sinking-fund provision, maintenance and replacement fund provision

(e) ABC’s CFO believes that an extendible reset bond is the same as a floating-rate
asset because the coupon rate may reset annually or even more frequently.

Critique this statement.

Commentary on Question:
Not many candidates understand how extendible reset bond works.

Extendible reset bond is different from a floating-rate asset


• A floating-rate asset reset coupon with a fixed spread over the reference
rate
• An extendible reset bond coupon rate reflects market condition and credit
spread at time of reset

APM Spring 2013 Solutions Page 47

You might also like